You are on page 1of 64

IDMP TEST SERIES-2023

High Yielding PRELIMS TEST SERIES

Test-27
Economics
Topic Covered:
❖ Indian financial market and insurance in India
❖ Banking in India
❖ Security market in India
❖ External sector in India

IDMP-2023
Contact us :info@onlyias.com

OnlyIAS Nothing Else Visit :dpp.onlyias.in


Contact : +91-7007 931 912

Q.1) Which of the following things may happen if b) 2 only


domestic currency is depreciating in context of c) Both 1 and 2
foreign currency borrowings? d) Neither 1 nor 2
1) Debt service liability would decrease.
2) More domestic currency would be required to buy Q.5) Which of the following statements about trends
the same amount of foreign exchange. in India’s overall trade are correct?
3) Could create corporate distress in the domestic 1) India’s share in world exports has increased from
market. 1991 to 2018.
Select the correct answer from the codes given below: 2) India’s services trade has been a major driver of its
a) 1 and 2 only exports due to its high growth and services trade
b) 1 and 3 only surplus has been financing most of India’s
c) 2 and 3 only merchandise trade deficit.
d) All of the above 3) Government has set overall export target of $1
trillion exports by 2027.
Q.2) Which of the following statement/s about the Select the correct answer from the codes given below:
‘debt overhang’ problem is/are correct? a) 1 and 2 only
1. It refers to a situation where all accumulated b) 2 and 3 only
profits over the years gets used up in repaying the c) 1 and 3 only
accumulated debt. d) All of the above
2. Debt overhang reduces social welfare.
Select the correct answer from the codes given below: Q.6) Which of the following statements about Deposit
a) 1 only Insurance and Credit Guarantee Corporation (DICGC)
b) 2 only is/are correct?
c) Both 1 and 2
1) It registers a bank as insured immediately and
d) Neither 1 nor 2
automatically on issuance of banking license.
Q.3) About Remission of Duties or Taxes on Export 2) For all insured banks, Payment of deposit
Product (RoDTEP), which of the following statements insurance premium to DICGC is compulsory
is/are correct? 3) Deposit insurance cover is ₹1 lakh per depositor
1) RoDTEP was formed to replace the existing MEIS per bank
(Merchandise Exports from India Scheme) Select the correct answer from the codes given below:
2) Duties/ taxes/ levies, at the Central, State & local a) 1 and 2 only
level, borne on the exported and imported
b) 1 and 3 only
product, including prior stage cumulative indirect
taxes on goods & services used in production are c) 2 and 3 only
refunded. d) All of the above
Select the correct answer from the codes given below:
a) 1 only Q.7) Under the Economic Capital Framework or ECF
b) 2 only the amount of surplus that the RBI must transfer to
c) Both 1 and 2 the Centre is determined by
d) Neither 1 nor 2
1) Realized equity
Q.4) About the Export Promotion Capital Goods 2) Economic capital
Scheme, which of the following statement/s is/are Select the correct answer from the codes given below:
correct? a) 1 only
1) Imports of capital goods are allowed duty free, b) 2 only
subject to an export obligation. c) Both 1 and 2
2) The Indian Government with the help of this d) Neither 1 nor 2
scheme offers incentives and financial support to
the exporters.
Q.8) Which of the following statements about
Select the correct answer from the codes given below:
Targeted Long-Term Repo Operations are correct?
a) 1 only
DPP 2023_DAY 188 1
Contact us :info@onlyias.com

OnlyIAS Nothing Else Visit :dpp.onlyias.in


Contact : +91-7007 931 912

1) They are conducted by the RBI to ensure adequate 1) It is a framework that prescribes higher standards
liquidity for a short period for the entire economy. of corporate governance for listed companies than
2) The TLTROs are exempted from regulatory those required by regulations.
requirement of the cash reserve ratio for the 2) All NSE-listed companies need to adopt it
equivalent of incremental credit disbursed by mandatorily.
banks as loans. Select the correct answer from the codes given below:
3) Under TLTROs, the RBI will be auctioning funds to a) 1 only
commercial banks up to a certain amount based on b) 2 only
the repo rate. c) Both 1 and 2
Select the correct answer from the codes given below: d) Neither 1 nor 2
a) 1 and 2 only
b) 2 and 3 only Q.13) About Anchor Investor, which of the following
c) 1 and 3 only statements is/are incorrect?
d) All of the above 1) Anchor investors are institutional investors who
are invited to subscribe the shares before the Initial
Q.9) GN Bajpai committee was seen in news, is related Public Offers (IPO) are made available to the public
to as per Sebi regulations.
a) Open market operations of RBI 2) Each investor is required to invest at least Rs 10
b) To examine India’s security market crore and the funds are subject to a 45-day lock-in
c) Tracking Outcomes of Insolvency and Bankruptcy period.
Code, 2016 Select the correct answer from the codes given below:
d) To suggest reforms in India’s logistic sector a) 1 only
b) 2 only
Q.10) Recently T+1 Settlement Cycle was seen in c) Both 1 and 2
news, is related to d) Neither 1 nor 2
a) Banking
Q.14) About Impact Investment which of the
b) Security market
following statements is/are correct?
c) Insurance sector
1) Impact investing is a general investment strategy
d) Pension
that seeks to create a positive atmosphere in the
market to inculcate confidence in other investors.
Q.11) Which of the following statements is/are
2) The aim of impact investing is to use money and
correct about Floating Rate Funds?
investment capital for positive social results.
1) Floating rate funds are those funds whose value
Select the correct answer from the codes given below:
changes with the demand and supply of such funds
a) 1 only
in the market
b) 2 only
2) Floating rates insulate investors from losses due to
c) Both 1 and 2
drop in its rates.
d) Neither 1 nor 2
Select the correct answer from the codes given below:
a) 1 only
Q.15) Consider the following statements regarding
b) 2 only
Capital Adequacy Ratio (CAR) :
c) Both 1 and 2
1. Capital Adequacy Ratio is calculated by summing a
d) Neither 1 nor 2
bank’s tier 1 capital and tier 2 capitals and dividing
the total by its total risk-weighted assets.
Q.12) Which of the following statements about NSE
Prime is/are correct?

DPP 2023_DAY 188 2


Contact us :info@onlyias.com

OnlyIAS Nothing Else Visit :dpp.onlyias.in


Contact : +91-7007 931 912

2. Tier 2 capital, also called core capital, consists of 3. The Bill includes capital redemption and annuity
ordinary share capital, equity capital, audited certain within the definition general insurance
revenue reserves, and intangible assets. business.
Which of the statements given above is /are correct? Which of the statements given above is/ are correct?
a) 1 only a) 1 only
b) 2 only b) 2 and 3 only
c) Both 1 and 2 c) 1 and 3 only
d) Neither 1 nor 2 d) 1, 2 and 3

Q.16) Consider the following statements regarding Q.19) Consider the following statements regarding
PM- Fasal Bima Yojana: Social Stock Exchange (SSE):
1. PM- Fasal Bima yojana is launched by the Ministry 1. It lists only the non-profit organisations (NPO) on
of Agriculture but administered by IRDAI under the stock exchanges.
ministry of Finance. 2. It will allow non-profit organisations to directly list
2. The Scheme shall be implemented on an ‘Area through issuance of bonds in the form of zero coupon
Approach basis’. or zero principal bonds.
3. The scheme is mandatory for loanee farmers . 3. Religious organisations and affordable housing
Which of the statements given above is/are correct? companies will be eligible to be identified as a social
a) 1 only enterprise.
b) 1 and 3 only Which of the statements given above is/are incorrect?
c) 2 only a) 1 and 2 only
d) 1, 2 and 3 b) 2 and 3 only
c) 1 and 3 only
Q.17) Which of the following statements are correct d) All of the above
about Central Bank Digital Currency (CBDC) ?
1. It enables timely transmission of monetary policy Q.20) Consider the following statements regarding
2. It can be a safer Substitute of bank deposit Sweat Equity:
3. In distress situation it help to reduce Bank Runs 1. It is a monetary contribution that the individuals or
4. It can increase deposit and retail lending rates. founders of a company make towards the company.
5. Its safer than present existing payment structure 2. Sweat equity allows companies to raise funds
Choose the correct option using the code given below: without raising debt levels.
a) 1, 2, and 3 only Which of the statements given above is/are Incorrect?
b) 1, 2, 4 and 5 only a) 1 only
c) 1, 3, 4 and 5 only b) 2 only
d) 1, 2, and 5 only c) Both 1 and 2
d) Neither 1 nor 2
Q.18) Consider the following statements regarding
General Insurance Amendment Bill,2021: Q.21) Consider the following statements regarding
1. The bill seeks to provide for a greater private sector Domestic Systemically Important Insurers (DSIIs):
participation in the public sector insurance 1. D-SIIs refer to insurers of such size, market
companies. importance and domestic and global
2. The bill ensures the shareholding of the central interconnectedness whose distress or failure would
government in the specified insurers must be at cause a significant dislocation in the domestic financial
least 51%. system.

DPP 2023_DAY 188 3


Download From - https://upscmaterial.online/

Contact us :info@onlyias.com

OnlyIAS Nothing Else Visit :dpp.onlyias.in


Contact : +91-7007 931 912

2. D-SIIs are subjected to very few regulatory measures 1. The fifth Schedule has provision for the creation of
to deal with the systemic risks and moral hazard issues. Autonomous District Councils and Regional Councils.
3. Currently Life Insurance Corporation of India (LIC) 2. The ADCs are empowered with civil and judicial
has been recognised as one of the Domestic powers.
Systemically Important Insurers (D-SIIs) for 2020-21 by 3. The president of India is empowered to reorganize
SEBI. or alter the boundaries of the tribal areas.
Which of the statements given above is/are Incorrect? Which of the statements given above is/are Correct?
a) 1 only a) 1 and 2 only
b) 2 only b) 2 only
c) 2 and 3 only c) 3 only
d) None of the above d) 1 and 3 only

Q.22) Consider the following statements regarding Q.25) Consider the following statements regarding
Euro Green Bond: Contempt of Court:
1. A green bond is a type of fixed-income instrument 1. The prior consent of the Attorney General (AG) of
that is specifically earmarked to raise money for India is required to suo motu initiate the inherent
climate and environmental projects. contempt powers of the Supreme Court.
2. HDFC Bank was the first bank to issue green bonds in 2. According to The Contempt of Courts Act, 1971,
India. contempt of court can either be civil contempt or
3. Power Finance Corporation Ltd (PFC) issued India’s criminal contempt.
first ever Euro denominated Green bond. 3. The motion before the court for initiating a case of

e
Which of the statements given above is/are Incorrect? criminal contempt can be brought by a private citizen

n
a) 1 only without the consent of the Attorney General or the

li
b) 2 only Advocate General.

n
c) 2 and 3 only Which of the statements given above is/are Correct?
d) None of the above a) 2 only o
b) 1 and 2 only
l.
Q.23) Consider the following statements regarding c) 1 and 3 only
a

Narcotic Drugs and Psychotropic Substances Act d) 1, 2 and 3 only


ri

(NDPS) :
te

1. Under the Act, property acquired by a person from Q.26) Consider the following statements :
drug-related offences, who has been convicted under 1. The legislature of a state may adopt any one or more
a

the Act can be seized, frozen and forfeited by the of the languages in use in the state or Hindi as the
m

government. official language of that state.


2. All the offences under the NDPS Act are non-bailable. 2. Under Article 343(1) of the Constitution, the official
c

3. Cannabis (hemp) and Bhang are covered under the language of the Union shall be Hindi in Devanagari.
s

NDPS Act. 3. The Constitution of India has given hindi language a


p

Which of the statements given above is/are correct? national status.


a) 1 only Which of the statements given above is/are correct?
.u

b) 2 and 3 only a) 1 only


w

c) 1 and 2 only b) 2 only


d) All of the above c) 1 and 3 only
w

d) 1 and 2 only
Q.24) Consider the following statements regarding
w

Autonomous District Councils (ADC):

DPP 2023_DAY 188 4


https://upscmaterial.online/
Download From - https://upscmaterial.online/

Contact us :info@onlyias.com

OnlyIAS Nothing Else Visit :dpp.onlyias.in


Contact : +91-7007 931 912

Q.27) Consider the following statements c) Both 1 and 2


regarding“Scheduled Castes” : d) Neither 1 nor 2
1. Article 342 of the Constitution prescribes the
procedure for recognising castes as “Scheduled Q.30) Consider the following Constitutional
Castes”. Amendments Acts (CAA) and their Relevance:
2. Any change (inclusion or exclusion) to the list of 1. The Constitution (102th Amendment) Act :
Scheduled Castes specified in the President's order can Reservations for Economic Weaker Section
only be made by Parliament by law. 2. The Constitution (103th Amendment) Act:
3. Proposals agreed by the National Commission for Constitutional status to the National Commission for
Scheduled Castes are introduced directly as a Bill in Backward Classes
Parliament for inclusion or exclusion of any community 3. The Constitution (104th Amendment) Act : Extended
to the list of Scheduled Castes. the reservation of seats for SCs and STs in the Lok
Which of the statements given above is/are Incorrect? Sabha and states assemblies
a) 1 only 4. The Constitution (105th Amendment) Act : Restore
b) 2 and 3 only the power of State governments to identify OBCs that
c) 1 and 3 only are socially and educationally backward
d) All of the above How many of the above pair is/are matched
correctly?
Q.28) Consider the following statements regarding a) Only one pair
Euthanasia : b) Only two pairs
1. Active Euthanasia involves simply stopping lifesaving c) Only three pairs

e
treatment or medical intervention with the consent of d) All four pairs

n
the patient or a family member or a close friend

li
representing the patient. Q.31) Which of the following act lays down

n
2. The Supreme Court of India legalised passive procedures for delimitation of constituencies and also
euthanasia in 2018, stating that it was a matter of provides for the allocation of seats in the parliament
o
‘living will’. and state legislature?
l.
Which of the statements given above is/are Incorrect? a) Representation of the People Act (RPA), 1950
a

a) 1 only b) Representation of People Act, 1951


ri

b) 2 only c) Representation of the People (Amendment) Act,


te

c) Both 1 and 2 1966


d) Neither 1 nor 2 d) Representation of the People (Amendment) Act,
a

2002
m

Q.29) Consider the following statements in context of


Preventive Detention: Q.32) Consider the following statements regarding
c

1. A detainee under preventive detention can have the the Co-operative Societies:
s

right of personal liberty guaranteed by Article 19 or 1. The state legislature shall provide for the reservation
p

Article 21. of one seat for the Scheduled Castes or the Scheduled
2. Under Article 22 (3), If a person is arrested or Tribes and two seats for women on the board of every
.u

detained under preventive detention, then the co-operative society.


w

protection against arrest and detention under Article 2. The 95th Constitutional Amendment Act (2011) gave
22 (1) and 22(2) shall not be available. constitutional status and protection to the co-
w

Which of the statements given above is/are correct? operative societies.


a) 1 only
w

b) 2 only

DPP 2023_DAY 188 5


https://upscmaterial.online/
Download From - https://upscmaterial.online/

Contact us :info@onlyias.com

OnlyIAS Nothing Else Visit :dpp.onlyias.in


Contact : +91-7007 931 912

3. The constitutional provisions related to cooperatives a) International Monetary Fund (IMF)


are incorporated in Fundamental Rights and Directive b) World Bank
Principles of State Policy. c) Asian Development Bank
Which of the statements given above is/are correct? d) International Development Association (IDA)
a) 1 and 2 only
b) 3 only Q.36) Consider the following statements regarding
c) 1 and 3 only Transboundary Protected Areas (TPA) :
d) None of the above 1. TPA is an area of land and/or sea that straddles one
or more borders between states, and whose
Q.33) Consider the following statements regarding constituent parts are dedicated to protection and
Swami Vivekananda: maintenance of biological diversity through legal or
1. Mahatma Gandhi called Vivekananda the “maker other effective means.
of modern India.” 2. India has TPAs with all the neighbouring countries.
2. "Karmyogin" , a journal written by Swami 3. It helps to maintain and strengthen ecosystem
Vivekananda. resilience in the face of climate change.
3. He was a major contributor towards the spread of Which of the statements given above is/are correct?
Yoga and Vedanta. a) 1 and 2 only
Which of the statements given above is/ are correct? b) 2 and 3 only
a) 1 only c) 1 and 3 only
b) 2 and 3 only d) All of the above
c) 3 only

e
d) 1, 2 and 3 Q.37) The National Park is essentially a network of

n
creeks and canals which are inundated with waters

li
Q.34) Consider the following statements regarding from rivers Brahmani, Baitarani, Dhamra and Patasala

n
G33 Group : forming a unique ecosystem.It is a Ramsar site and the
1. The group has advocated the creation of a "special second largest mangrove ecosystem of India.It is said
o
products" exemption, which would allow developing to house 70% of the country’s estuarine or saltwater
l.
countries to exempt certain products from tariff crocodiles, conservation of which was started way
a

reductions. back in 1975. Which of the following national park


ri

2. The G33 is a group of 33 developing and developed best describes the above statement?
te

nations which is also known as the Friends of Special a) Sundarbans National Park
Products in agriculture. b) The Gulf of Mannar Marine National Park
a

3. The peace clause protects a developing country’s c) Bhitarkanika National Park


m

food procurement programmes against action from d) Ranthambore National Park


WTO members in case subsidy ceilings(10%) are
c

breached. Q.38) Consider the following statements regarding


s

Which of the statements given above is/are correct? Martand Sun Temple:
p

a) 1 and 2 only 1. The Temple was built in the 8th century by the
b) 2 and 3 only third ruler of the Kakatiya Dynasty.
.u

c) 1 and 3 only 2. The Temple is devoid of any external influence in


w

d) All of the above its architecture.


3. The temple was destroyed by the orders of Muslim
w

Q.35) Extended Fund Facility (EFF) is a lending facility ruler Mohammad Bin Qasim.
to help members with balance of payments. Which of Which of the statements given above is/are incorrect?
w

the following organization provides this facility? a) 1 only

DPP 2023_DAY 188 6


https://upscmaterial.online/
Download From - https://upscmaterial.online/

Contact us :info@onlyias.com

OnlyIAS Nothing Else Visit :dpp.onlyias.in


Contact : +91-7007 931 912

b) 3 only d) 1, 2 and 3
c) 1 and 2 only Q.42) Consider the following statements regarding
d) 1, 2 and 3 ‘zombie ice’ :
1. It is referred to as dead or doomed ice is one that
Q.39) Consider the following statements regarding accumulates fresh snow while continuing to be part of
lumpy skin disease : the parent ice sheet.
1. It is caused by the lumpy skin disease virus (LSDV) 2. Such ice is committed to not melting away and
which is a zoonotic virus. decreasing sea levels.
2. It is a contagious vector-borne disease and usually Which of the statements given above is/are correct?
affects animals like cows and water buffaloes. a) 1 only
Which of the statements given above is/are correct? b) 2 only
a) 1 only c) Both 1 and 2
b) 2 only d) Neither 1 nor 2
c) Both 1 and 2
d) Neither 1 nor 2 Q.43) Consider the following statements regarding
Advanced Chemistry Cell ACC :
Q.40) Consider the following statements regarding 1. These are the new generation technologies that
One-Horned Rhino : can store electric energy either as electrochemical
1. The Great One-Horned Rhino is found in India and or as chemical energy and convert it back to
Indonesia. electric energy as and when required
2. It comes under Schedule I of wildlife protection 2. The Production Linked Incentive (PLI) scheme for

e
act,1972 and Endangered status on IUCN red list. manufacturers of Advanced Chemistry Cell (ACC)

n
3. The five rhino range nations (India, Bhutan, Nepal, battery storage, is launched under The Ministry of

li
Indonesia and Malaysia) have signed a declaration ‘The Science and Technology.

n
New Delhi Declaration on Asian Rhinos 2019’ for the Which of the statements given above is/are correct?
conservation and protection of the species. a) 1 only o
Which of the statements given above is/are Incorrect? b) 2 only
l.
a) 2 only c) Both 1 and 2
a

b) 3 only d) Neither 1 nor 2


ri

c) 1 and 2 only
te

d) 1 and 3 only Q.44) Consider the following statements regarding


Military exercises:
a

Q.41) Consider the following statements regarding 1. ‘Vostok 2022’ is a joint military exercise between
m

‘Kritagya Hackathon 3.0’ : Russian and Indian Military forces.


1. Kritagya Hackathon aims to promote potential 2. ‘Exercise Hand in Hand’ is a joint military exercise
c

technology solutions for enhancing farm between India and Indonesia.


s

mechanisation. 3. Garuda Shakti exercise is a joint military exercise


p

2. In the Hackathon students and faculty of any between India and China .
University across the country can participate. Which of the statements given above is/are incorrect?
.u

3. The Kritagya Hackathon is launched by The Indian a) 1 only


w

Council of Agriculture Research. b) 2 and 3 only


Which of the statements given above is/ are correct? c) 2 only
w

a) 1 and 3 only d) 1, 2 and 3


b) 2 and 3 only
w

c) 2 only

DPP 2023_DAY 188 7


https://upscmaterial.online/
Download From - https://upscmaterial.online/

Contact us :info@onlyias.com

OnlyIAS Nothing Else Visit :dpp.onlyias.in


Contact : +91-7007 931 912

Q.45) Consider the following statements regarding 2. The Troposphere shields our planet from harmful
INS Vikrant: solar and cosmic particle radiation, as well as
1. INS Vikrant is the largest naval ship designed and erosion of the atmosphere by the solar wind
built in India with collaboration of Russia . 3. The time of arrival and the speed of solar storms
2. The motto of the ship is ‘Jayema Sam Yudhi and solar activities in general cannot be predicted
Sprdhah’ taken from Rig Veda. in advance.
3. The ship is capable of operating an air wing Which of the statements given above is /are not
consisting of 30 aircraft. correct ?
Which of the statements given above is/are correct? a) 2 only
a) 1 only b) 3 only
b) 1 and 2 only c) 2 and 3 only
c) 2 and 3 only d) 1 and 3 only
d) 1, 2 and 3
Q.49) Consider the following statements regarding
Q.46) Which of the following statements correctly Rainbow Clouds:
explains ‘Dharmshala Declaration 2022’ recently seen 1. A rainbow cloud is a form of pileus cloud that is
in the news: usually formed over a nimbostratus cloud.
a) A declaration to save Ganga 2. The phenomenon of bright colours appearing on a
b) A declaration to save Himalayan Glaciers cloud is called cloud iridescence which is an optical
c) A declaration for tourism development phenomenon.
d) A declaration against Chinese aggression on border Which of the statements given above is/are correct?

e
a) 1 only

n
Q.47) Consider the following statements regarding b) 2 only

li
Quantum internet : c) Both 1 and 2

n
1. Classical computer data code takes the form of d) Neither 1 nor 2
either zeros or ones and quantum information is o
superimposed in both zeros and ones Q.50) Consider the following statements regarding
l.
simultaneously. Solomon Islands:
a

2. Classical networks apply phenomena, such as no- 1. Solomon Islands is a volcanic country located in the
ri

cloning, entanglement, and superposition which Pacific Ocean.


te

are not available to quantum internet networks. 2. It's part of the Kuril Islands group.
Which of the statements given above is/ are not 3. Solomon Islands are part of the Commonwealth of
a

correct ? Nations Group.


m

a) 1 only Which of the statements given above is/are correct?


b) 2 only a) 1 only
c

c) Both 1 and 2 b) 1 and 3 only


s

d) Neither 1 nor 2 c) 2 only


p

d) 2 and 3 only
Q.48) Consider the following statements regarding
.u

Geomagnetic storm : Q.51) Consider the following statements regarding


w

1. A geomagnetic storm is a major disturbance of Aridity Anomaly Outlook Index:


Earth's magnetosphere that occurs when there is 1. The Index is developed by the World
w

an exchange of energy from the solar wind into the Meteorological Organization.
space surrounding Earth. 2. The Index is computed for weekly or two-weekly
w

periods. For each period, the actual aridity for the

DPP 2023_DAY 188 8


https://upscmaterial.online/
Download From - https://upscmaterial.online/

Contact us :info@onlyias.com

OnlyIAS Nothing Else Visit :dpp.onlyias.in


Contact : +91-7007 931 912

period is compared to the normal aridity for that activity and industrial activity as principal
period. businesses.
3. The Index is applicable for summer season crops 2. NBFCs whose asset size is of ₹ 500 crore or more
but has no utility for winter season crops. are considered as systemically important NBFCs.
Which of the statements given above is/are correct? 3. Deposit Insurance and Credit Guarantee
a) 1 and 2 only Corporation facility is available to depositors of
b) 2 only NBFCs.
c) 1 and 3 only Which of the statements given above is /are correct?
d) 2 and 3 only a) 2 only
b) 1 only
Q.52) Consider the following statements regarding c) 1 and 3 only
Cervavac : d) 1, 2 and 3
1. It is a quadrivalent human papillomavirus (qHPV)
vaccine for the prevention of cervical cancer developed Q.55) Consider the following statements regarding
by WHO. Prompt Corrective Action (PCA) Framework:
2. Cervical cancer is a prevalent sexually transmitted 1. Prompt Corrective Action is a qualitative tool
infection. introduced by the Ministry of Finance under which
3. India accounts for about a fifth of the global cervical direct action is taken on weak banks to ensure the
cancer burden. financial health of a bank.
Which of the statements given above is/are correct? 2. The PCA framework is applicable only to
a) 1 and 2 only commercial banks and not extended to co-

e
b) 2 and 3 only operative banks, non-banking financial companies

n
c) 1 and 3 only (NBFCs) .

li
d) All of the above 3. A bank prohibited from expansion of

n
credit/investment portfolios under PCA will,
Q.53) Consider the following statements regarding however, be allowed to invest in government
o
hybrid propulsion system : securities/other high-quality liquid investments.
l.
1. It uses Hydroxyl-terminated polybutadiene (HTPB) as Which of the statements given above is/ are correct?
a

solid fuel and liquid nitrogen (LN) as the liquid fuel. a) 1 and 3 only
ri

2. The hybrid system is more efficient, ''greener'' and b) 3 only


te

safer to handle. c) 2 and 3 only


3. Unlike conventional solid motors, the hybrid d) 1, 2 and 3
a

technology does not permit restarting and throttling


m

capabilities on the motor. Q.56) Consider the following statements regarding


Which of the statements given above is/are Incorrect? Payments Banks :
c

a) 1 only 1. Suggested by the Nachiket Mor committee in 2013,


s

b) 3 only Payments Bank is a bank registered under Reserve


p

c) 2 and 3 only Bank of India Act, 1934 and licensed under Banking
d) 1 and 3 only Regulation Act, 1949.
.u

2. Payments Banks cannot accept demand deposits


w

Q.54) Consider the following statements regarding but can offer remittance services, mobile
non-bank financial institution NBFCs: purchases and other banking services
w

1. NBFCs, also known as nonbank financial 3. Securities and Exchange Board of India has allowed
institutions and have business with agriculture the payment banks to act as investment bankers.
w

Which of the statements given above is / are correct?

DPP 2023_DAY 188 9


https://upscmaterial.online/
Download From - https://upscmaterial.online/

Contact us :info@onlyias.com

OnlyIAS Nothing Else Visit :dpp.onlyias.in


Contact : +91-7007 931 912

a) 3 only 2. The RTGS method does not have a minimum


b) 2 only transfer limit ceiling over the amount.
c) 1 and 2 only 3. In NEFT , the settlement of funds happens in real
d) 1, 2 and 3 time, means they are processed as they are made
Which of the statements given above is/ are not
Q.57) Consider the following statements regarding correct?
Deposit Insurance and Credit Guarantee Corporation a) 1 only
(DICGC): b) 2 and 3 only
1. DICGC is a wholly owned subsidiary of the RBI c) 1 and 2 only
created under the DICGC Act, 1961 with Governor d) 1, 2 and 3
of RBI as its ex-officio Chairman.
2. DICGC (Amendment) Act, 2021 has provisions for Q.60) Consider the following statements regarding
penal interest in case of delay to ensure timely Money Multiplier :
repayment from banks to DICGC. 1. Money multiplier is the maximum limit to which
3. It covers all commercial banks, excluding Payment money supply can be affected by bringing about
Banks, Small Finance Banks and Regional Rural changes in the amount of money deposits.
Banks. 2. There is a direct relationship between the Money
Which of the statements given above is/ are not Multiplier and the legal reserve ratio, it means that
correct? if the reserve ratio is higher, then the money
a) 2 only multiplier will also be higher.
b) 2 and 3 only Which of the statements given above is /are correct ?

e
c) 1 and 3 only a) 1 only

n
d) 1, 2 and 3 b) 2 only

li
c) Both 1 and 2

n
Q.58) Consider the following statements regarding d) Neither 1 nor 2
Domestic Systemically Important Banks (D-SIBs): o
1. The Ministry of Finance places D-SIBs in
l.
appropriate buckets depending upon their Q.61) Consider the following statements regarding
a

Systemic Importance Scores (SISs). Money Supply:


ri

2. Due to their economic and national importance, 1. The total stock of money in circulation among the
te

the banks need to maintain a higher share of risk- public at a particular point of time is called money
weighted assets as tier-I equity. supply.
a

Which of the statements given above is /are not 2. M1 and M2 are known as narrow money whereas
m

correct? M3 and M4 are known as broad money.


a) 1 only 3. M4 is most liquid and easiest for transactions
c

b) 2 only whereas M1 is least liquid of all and the gradations


s

c) Both 1 and 2 are in increasing order of liquidity from M1 to M4.


p

d) Neither 1 nor 2 Which of the statements given above is /are not


correct ?
.u

Q.59) Consider the following statements regarding a) 1 only


w

National Electronic Funds Transfer (NEFT) vs Real- b) 2 and 3 only


Time Gross Settlement (RTGS) : c) 3 only
w

1. The NEFT mode is used when the transactions are d) 1, 2 and 3


of smaller values while RTGS is used in high-value
w

transactions.

DPP 2023_DAY 188 10


https://upscmaterial.online/
Download From - https://upscmaterial.online/

Contact us :info@onlyias.com

OnlyIAS Nothing Else Visit :dpp.onlyias.in


Contact : +91-7007 931 912

Q.62) Consider the following statements regarding 3. Foreign institutional investors (FIIs) are companies
Payments Bank : based outside of India that make investment
1. The Payments Bank is proposed to be registered as proposals in the country.
a public limited company under the Companies Act Which of the statements given above is/ are correct?
of 2013, and licenced under the Banking a) 2 only
Regulation Act of 1949. b) 3 only
2. They can’t advance loans or issue credit cards, and c) 1 and 3 only
cannot accept deposits from the Non-Resident d) 1, 2 and 3
Indians (NRIs)
3. It is not mandatory for the payments bank to Q.65) Consider the following statements regarding
maintain Cash Reserve Ratio (CRR) with the Infrastructure investment trusts (InvITs) :
Reserve Bank of India. 1. Infrastructure investment trusts (InvITs) are like
Which of the statements given above is /are correct? mutual funds that pool money from investors and
a) 1 and 2 only are regulated by RBI.
b) 2 and 3 only 2. An InvIT is established as a trust settled by the
c) 1 and 3 only sponsor under the Indian Trusts Act, 1882 and the
d) 1,2 and 3 trust deed registered in India under the
Registration Act, 1908.
Q.63) Consider the following statements regarding 3. The investment must be made only in completed
Small finance banks (SFBs) : and revenue-generating projects and not in under-
1. They are established as public limited companies construction projects

e
under the Companies Act, 2013 that can provide Which of the statements given above is /are correct?

n
basic banking service of acceptance of deposits and a) 2 only

li
lending. b) 3 only

n
2. Existing non-banking financial companies (NBFC), c) 1 and 3 only
microfinance institutions (MFI) and local area d) 1, 2 and 3 o
banks (LAB) can apply to become small finance
l.
banks Q.66) It is a basket of securities that trade on an
a

3. Foreign shareholding will not be allowed in these exchange, just like a stock. It reflects the composition
ri

banks as per the rules for FDI in private banks in of an Index, like BSE Sensex. Its trading value is based
te

India. on the Net Asset Value (NAV) of the underlying stocks


Which of the statements given above is/ are correct? (such as shares) that it represents. Its share prices
a

a) 1 and 2 only fluctuate all day as it is bought and sold. Which of the
m

b) 3 only following financial instruments are referred to in the


c) 2 and 3 only above lines ?
c

d) 1, 2 and 3 a) Mutual funds


s

b) Exchange Traded Fund


p

Q.64) Consider the following statements regarding c) Alternative investment fund


the types of foreign investments: d) Hedge funds
.u

1. Foreign Portfolio Investments (FPI) includes


w

investments in high-yielding assets such as the Q.67) Which of the following best describes the term
plant and machinery of a business “Yield Inversion” with reference to the Indian
w

2. Foreign Direct Investments (FDI) Investors can Economy ?


quickly depart from the country as they invest in a) A phenomenon where the yield of the government
w

stocks and bonds, which are liquid. bond rises .

DPP 2023_DAY 188 11


https://upscmaterial.online/
Download From - https://upscmaterial.online/

Contact us :info@onlyias.com

OnlyIAS Nothing Else Visit :dpp.onlyias.in


Contact : +91-7007 931 912

b) A phenomenon where the yield of the government company that sells the bond. These bonds protect the
bond falls. beneficiary against acts or events that impair the
c) When the yield on a bond with a longer term underlying obligations of the principal. Which of the
becomes lower than the yield on a bond with a following types of bonds is mentioned in the above
shorter term lines ?
d) When the yield on a bond with a shorter term a) Additional Tier – 1 (AT-1 Bonds) Bonds
becomes lower than the yield on a bond with a b) Social impact bond (SIB)
longer term c) Surety bonds
d) Masala bonds
Q.68) Financial market is classified into money market
and capital market. Which of these are Capital market Q.71) Which one of the following best describes the
instruments? term “ Certificate of Deposit” ?
1. Commercial Paper a) An unsecured instrument issued in the form of a
2. Certificate of Deposit promissory note to enable the corporate
3. Bonds borrowers to raise short-term funds
4. Debentures b) It is a promissory note issued by the RBI to meet
Select the correct answer using the code given below: the short-term requirement of funds.
a) 1 and 2 only c) The short-term instruments issued by Commercial
b) 2, 3 and 4 only Banks and Special Financial Institutions (SFIs),
c) 3 and 4 only which are freely transferable from one party to
d) 1, 2, 3 and 4 another.

e
d) An instrument in writing containing an

n
Q.69) Consider the following statements regarding unconditional order, signed by the maker, directing

li
the Sovereign Gold Bonds (SGBs) : a certain person to pay a certain sum of money

n
1. They are substitutes for holding physical gold in only to the bearer of the instrument
which Investors have to pay the issue price in cash o
and the bonds will be redeemed in gold on Q.72) Consider the following statements regarding
l.
maturity. the Collective Investment Scheme :
a

2. The Bonds are issued in denominations of one 1. It is a scheme under which the contributions made
ri

gram of gold and in multiples thereof, and there is by the investors are pooled and the units of the
te

no minimum or maximum investment limit of schemes are listed on a stock exchange


subscription . 2. There are no rules regarding a minimum number of
a

3. These securities are eligible to be used as collateral investors or minimum subscription amount.
m

for loans from banks, financial Institutions and Which of the statements given above is /are not
Non-Banking Financial Companies (NBFC) correct?
c

Which of the statements given above is /are not a) 1 only


s

correct? b) 2 only
p

a) 1 and 3 only c) Both 1 and 2


b) 2 and 3 only d) Neither 1 nor 2
.u

c) 1 and 2 only
w

d) 1, 2 and 3 Q.73) Which of the following best describes the term


“Hedging” with reference to the Indian economy?
w

Q.70) This bond is defined as a three-party agreement a) A financial derivative that allows one investor to
that legally binds together a principal who needs the swap or balance their credit risk with that of
w

bond, an obligee who requires the bond, and a another.

DPP 2023_DAY 188 12


https://upscmaterial.online/
Download From - https://upscmaterial.online/

Contact us :info@onlyias.com

OnlyIAS Nothing Else Visit :dpp.onlyias.in


Contact : +91-7007 931 912

b) A risk management strategy employed to offset Q.76) Consider the following statements regarding
losses in investments by taking an opposite the Depositary receipt (DR):
position in a related asset. 1. A depository receipt is a negotiable financial
c) The buying of securities primarily to earn a regular instrument issued by a bank to reflect the security
income from such investment and possibly make of a foreign company's publicly traded securities
some long-term gain on account of price rise in that are exchanged on a local stock exchange
future 2. An Indian Depository Receipt (IDR) is a financial
d) The act of conducting a financial transaction that instrument that allows an Indian company to raise
has substantial risk of losing value but also holds funds in other countries .
the expectation of a significant gain. Which of the statements given above is /are not
correct ?
Q.74) Consider the following statements regarding a) 1 only
Treasury Bills : b) 2 only
1. Treasury bills or T-bills, which are money market c) Both 1 and 2
instruments, are long term debt instruments with d) Neither 1 nor 2
maturities higher than one year , issued by the
Government of India. Q.77) Consider the following statements regarding
2. Treasury bills are zero coupon securities that pay the Masala Bonds :
regular interests 1. Masala bonds are Dollar-denominated bonds
3. Banks give treasury bills to the RBI to get money issued in India by a corporate and Indian entity.
under repo and they can also keep it to fulfil their 2. The issuers have to deal with the transfer risk

e
Statutory Liquid Ratio (SLR) requirements. because of the fluctuations in the currency

n
Which of the statements given above is /are correct? 3. Masala Bond can be issued by private and

li
a) 2 only government businesses as well.

n
b) 3 only Which of the statements given above is /are correct ?
c) 1 and 3 only a) 1 and 2 only o
d) 1, 2 and 3 b) 3 only
l.
c) 2 and 3 only
a

Q.75) Consider the following statements regarding d) 1, 2 and 3


ri

the Chit Funds :


te

1. Chit funds are part of the Central List of the Indian Q.78) Consider the following statements regarding
Constitution, according to the Supreme Court, Financial Inclusion Index :
a

which classifies them as contracts. 1. It is a composite index to capture the extent of


m

2. Under the Chit Fund Act of 1962, businesses can be financial inclusion across the country published
registered and regulated only by the Central annually by the NITI Ayog
c

Government. 2. The index comprises three parameters including


s

3. Ponzi schemes are investment operations that pay access, usage and quality.
p

returns to old investors from the money garnered Which of the statements given above is/ are correct ?
from new investors. a) 1 only
.u

Which of the statements given above is /are not b) 2 only


w

correct ? c) Both 1 and 2


a) 1 and 2 only d) Neither 1 nor 2
w

b) 3 only
c) 2 and 3 only Q.79) Consider the following statements regarding
w

d) 1, 2 and 3 the National Payments Corporation of India

DPP 2023_DAY 188 13


https://upscmaterial.online/
Download From - https://upscmaterial.online/

Contact us :info@onlyias.com

OnlyIAS Nothing Else Visit :dpp.onlyias.in


Contact : +91-7007 931 912

(NPCI) : 1. Nationalisation of the 14 biggest commercial banks


1. The National Payments Corporation of India (NPCI) 2. Establishment of National Bank for Agriculture and
is an wholly owned subsidiary of Reserve Bank of RuralDevelopment (NABARD)
India (RBI) to operate the retail payments and 3. Formation of Narasimham Committee II
settlement systems in India 4. Nationalization of RBI
2. It has been incorporated as a “Not for Profit” Which of the following describes the correct
Company under the provisions of Section 25 of chronological sequence of above events
Companies Act 1956 to offer improved a) 4 - 1 - 2 - 3
infrastructure for the entire banking industry to b) 4 - 2 - 1 - 3
create a robust physical and digital payment and c) 1 - 4- 3 - 2
settlement system. d) 2 - 4 - 1 - 3
Which of the statements given above is/ are correct ?
a) 1 only Q.83) Consider the following statements regarding
b) 2 only India Post Payments Bank (IPPB) :
c) Both 1 and 2 1. It was launched in 2018 under the Department of
d) Neither 1 nor 2 Financial Services, Ministry of Finance and its 100%
equity is owned by the Government of India.
Q.80) Which of the following laws deals with Wilful 2. Fincluvation will be a permanent platform of IPPB,
Defaulters i.e. the entities that do not repay money which is a joint initiative to collaborate with the
despite having the ability to do so? Fintech Startup community to co-create and
1. The Fugitive Economic Offenders Act 2018 innovate solutions for financial inclusion.

e
2. Companies Act of 2013 Which of the statements given above is/ are correct ?

n
3. Banking Regulation Amendment Act of 2017 a) 1 only

li
Select the correct answer using the code given below: b) 2 only

n
a) 1 and 2 only c) Both 1 and 2
b) 1 and 3 only d) Neither 1 nor 2 o
c) 2 and 3 only
l.
d) 1, 2 and 3 Q.84) Which of the following reports/ indices are
a

released by RBI ?
ri

Q.81) Which of the following best describes the term 1. Financial Stability Report
te

“Shadow Banking” with reference to Indian economy: 2. Digital Payments Index


a) A type of direct banking that operates exclusively 3. Report on Currency and Finance (RCF)
a

online without traditional physical branch Select the correct answer using the code given below:
m

networks a) 1 only
b) A corporate structure which isolates illiquid and b) 1 and 2 only
c

high risk assets held by a bank or a financial c) 2 and 3 only


s

organisation, or perhaps a group of banks or d) 1, 2 and 3


p

financial organisations.
c) A term used to describe bank-like activities (mainly Q.85) Consider the following statements regarding
.u

lending) that take place outside the traditional Non-Performing Assets:


w

banking sector 1. The loans on which interest or installment of


d) The banks that have a specific purpose, focused on principal remain overdue for a period of more than
w

a particular subset of the population 90 days from the end of a particular quarter is
called a Non-performing Asset.
w

Q.82) Consider the following events:

DPP 2023_DAY 188 14


https://upscmaterial.online/
Download From - https://upscmaterial.online/

Contact us :info@onlyias.com

OnlyIAS Nothing Else Visit :dpp.onlyias.in


Contact : +91-7007 931 912

2. For short duration crop agriculture loans such as 2. Legal tender is money that is legally valid for the
paddy, Jowar, Bajra etc, if the loan (installment / payment of debts and that must be accepted for
interest) is not paid for 180 days , it would be that purpose when offered.
termed as a NPA. Which of the statements given above is /are not
3. SARFAESI Act 2002, has provisions for the banks to correct?
take legal recourse to recover their dues. a) 1 only
Which of the statements given above is /are correct? b) 2 only
a) 1 and 2 only c) Both 1 and 2
b) 2 and 3 only d) Neither 1 nor 2
c) 1 and 3 only
d) 1, 2 and 3 Q.89) V Subrahmanyam committee (1997), Usha
Thorat committee (2004), Y H Malegam committee
Q.86) With reference to the Indian economy , which (2013) and Bimal Jalan committee were constituted to
of the following best defines the term “Provisioning review which of the following?
Coverage Ratio” ? a) Disinvestment
a) The proportion of highly liquid assets held by b) Tax reform
financial institutions, to ensure their ongoing c) Economic capital framework (ECF)
ability to meet short-term obligations. d) Small-scale industries
b) The percentage of funds that a bank sets aside for
losses due to bad debts. Q.90) A central bank balance sheet is a reflection of its
c) The fraction of the deposits scheduled banks must various functions, particularly its role as a monetary

e
keep with RBI authority and as banker to the Government and

n
d) The percentage of a bank's total deposits that it banks. The structure of assets and liabilities of the

li
needs to maintain as liquid cash. Reserve Bank are, more or less, in line with the

n
balance sheet followed by most central banks. Which
Q.87) Consider the following statements regarding of the following constitutes the liability of RBI ?
o
Currency-deposit ratio : 1. Notes Issued and Notes in Circulation
l.
1. The currency deposit ratio (cdr) is the ratio of 2. Deposits of State Governments
a

money held by the public in currency to that they 3. Loans and Advances to SCBs, State Co-operative
ri

hold in bank deposits Banks


te

2. Currency-deposit ratio holds a direct proportion Select the correct answer using the code given below
relationship with the money supply. :
a

Which of the statements given above is /are correct ? a) 1 and 2 only


m

a) 1 only b) 2 only
b) 2 only c) 1 and 3 only
c

c) Both 1 and 2 d) 1, 2 and 3


s

d) Neither 1 nor 2
p

Q.91) Consider the following statements regarding


Q.88) Consider the following statements regarding Development Finance Institutions :
.u

monetary system: 1. In India, the first Development Finance Institutions


w

1. Fiat money is currency that is backed by a physical was operationalized in 1948 with the setting up of
commodity, such as gold or silver, and the value of the Industrial Development Bank of India
w

fiat money is derived from the worth of a 2. DFIs do not accept deposits from people but they
commodity backing it. raise funds by borrowing funds from governments
w

and by selling their bonds to the general public

DPP 2023_DAY 188 15


https://upscmaterial.online/
Download From - https://upscmaterial.online/

Contact us :info@onlyias.com

OnlyIAS Nothing Else Visit :dpp.onlyias.in


Contact : +91-7007 931 912

Which of the statements given above is/ are not a) 1 and 2 only
correct? b) 1 and 3 only
a) 1 only c) 2 only
b) 2 only d) 1, 2 and 3
c) Both 1 and 2
d) Neither 1 nor 2 Q.95) Consider the following statements regarding
Major Stock Exchanges in India :
Q.92) Consider the following statements regarding 1. National Stock Exchange (NSE), which was
National Asset Reconstruction Company Ltd. (NARCL) established in 1875, is Asia’s oldest stock exchange
: and it was the first stock exchange in India
1. NARCL has been incorporated under the 2. The RBI is the regulatory authority under Section 3
Companies Act and has applied to the Reserve of the RBI Act 1934, to safeguard the interests of
Bank of India for a license as an Asset investors in securities, promote the development
Reconstruction Company (ARC). of and regulate the securities market.
2. This new entity is being created in collaboration Which of the statements given above is /are not
with both public and private sector banks. correct?
Which of the statements given above is/ are correct? a) 1 only
a) 1 only b) 2 only
b) 2 only c) Both 1 and 2
c) Both 1 and 2 d) Neither 1 nor 2
d) Neither 1 nor 2

e
Q.96) Consider the following statements regarding

n
Q.93) The effect of inflation is not distributed evenly Trade Receivables Discounting System (TreDs):

li
in the economy. There are chances of hidden costs for 1. TReDS is an electronic platform for facilitating the

n
different goods and services in the economy. Which financing of trade receivables of Micro, Small and
of the following sections of people will benefit from Medium Enterprises (MSMEs) through multiple
o
the inflation? financiers authorised under the Payment and
l.
1. Creditor/lender Settlement Systems (PSS) Act, 2007
a

2. Pensioner 2. Corporates, Government Departments, PSUs and


ri

3. Debtor/Borrower any other entity can participate as sellers as well as


te

Select the correct answer using the code given below: buyers in TReDS
a) 2 only Which of the statements given above is/ are correct ?
a

b) 3 only a) 1 only
m

c) 2 and 3 only b) 2 only


d) 1, 2 and 3 c) Both 1 and 2
c

d) Neither 1 nor 2
s

Q.94) Consider the following statements regarding


p

the Debentures : Q.97) Consider the following statements regarding


1. It is a type of debt instrument that is not secured International Financial Services Centres Authority
.u

by physical assets or collateral. (IFSCA):


w

2. Debenture holders have rights to vote in the 1. IFSCA is a non statutory authority established by
company’s general meetings of shareholders. the Indian Government , to develop and regulate
w

3. They are often easy to sell in stock exchanges and financial services, financial products and financial
they contain less risk than equities. institutions in the International Financial Services
w

Which of the statements given above is/ are correct ? Centre (IFSC)

DPP 2023_DAY 188 16


https://upscmaterial.online/
Download From - https://upscmaterial.online/

Contact us :info@onlyias.com

OnlyIAS Nothing Else Visit :dpp.onlyias.in


Contact : +91-7007 931 912

2. An International Financial Services Centre (IFSC) is Q.100) Consider the following statements regarding
a jurisdiction that provides world class financial the differences between Primary Market Secondary
services exclusively to non residents, in the Market :
domestic currency of the location where the IFSC is 1. Primary Market is that segment of the capital
located. market which deals with new securities which is
Which of the statements given above is /are correct ? also known as ‘new issues market’
a) 1 only 2. The secondary market, or otherwise called the
b) 2 only stock market, is a place where the securities are
c) Both 1 and 2 traded between investors, after their issue to the
d) Neither 1 nor 2 public in the primary market
3. In a Secondary market, the company is directly
Q.98) Consider the following statements regarding involved in the transaction whereas, in the Primary
the Insurance sector in India: market, there is no involvement of the company.
1. Both the Life Insurance and the Non-life Insurance Which of the statements given above is /are not
are regulated by the IRDAI (Insurance Regulatory correct ?
and Development Authority of India) which was a) 1 only
setup on the recommendations of R N Malhotra b) 2 and 3 only
committee. c) 3 only
2. India is at par with international average in terms d) 1 and 3 only
of insurance penetration for life insurance and in
the non-life insurance sector.

e
Which of the statements given above is /are not

n
correct?

li
a) 1 only

n
b) 2 only
c) Both 1 and 2 o
d) Neither 1 nor 2
l.
a

Q.99) What do you mean by the term “Blue Chip


ri

Stocks”, with reference to the Indian economy ?


te

a) The shares of very large and well-recognised


companies with a long history of sound financial
a

performance
m

b) The shares of companies mitigating climate change


through the conservation and restoration of
c

coastal and marine ecosystems.


s

c) The shares of companies those that reach a


p

valuation of $1 billion without being listed on the


stock market
.u

d) The shares of companies engaged in the design and


w

fabrication of semiconductors and semiconductor


devices.
w
w

DPP 2023_DAY 188 17


https://upscmaterial.online/
Download From - https://upscmaterial.online/

Contact us :info@onlyias.com

OnlyIAS Nothing Else Visit :dpp.onlyias.in


Contact : +91-7007 931 912

Q.1) Ans: C growth and a degradation of living standards from


reduced funds to spending in critical areas such as
Exp: healthcare, education, and infrastructure.
• Statement 1 is incorrect and Statements 2 and
3 are correct: A sharp depreciation in local Q.3) Ans: A
currency would mean corresponding increase Exp:
in debt service liability, as more domestic ● Statement 1 is correct: RoDTEP stands for
currency would be required to buy the same Remission of Duties and Taxes on Export
amount of foreign exchange for debt service Products. It is a new scheme that is applicable
payments. This would lead to erosion in profit with effect from January 1st, 2021, formed to
margin and have ‘mark-to-market’ replace the existing MEIS (Merchandise
implications for the corporate. There would Exports from India Scheme).
also be a ‘debt overhang’ problem, as the ● Statement 2 is incorrect: Scheme’s objective is
volume of debt would rise in local currency to refund, currently un-refunded:
terms. Together, these factors could create 1. Duties/ taxes/ levies, at the Central, State &
corporate distress, especially because the local level, borne on the exported product,
rupee tends to depreciate precisely when the including prior stage cumulative indirect taxes
Indian economy is also under stress, and on goods & services used in production of the
corporate revenues and margins are under exported product, and
pressure. 2. Such indirect Duties/ taxes/ levies in respect of
distribution of exported products.
Q.2) Ans: B • It may be noted that rebate under the Scheme
Exp: shall not be available in respect of duties and
• Statement 1 is incorrect: It refers to a situation

e
taxes already exempted or remitted or
where all current income gets used up in credited.

n
repaying the accumulated debt, leaving little • RoDTEP is going to give a boost to Indian

li
incentives to invest either in physical or human
exports by providing a level playing field to

n
capital.
• Statement 2 is correct: Debt overhang leads to domestic industry abroad.
abandonment of beneficial investment and
o
• (The 2nd statement is incorrect because there
l.
hence reduces social welfare. is no tax exemption of imports in this scheme.
a

This scheme deals with exports only and its


Extra Edge by OnlyIAS
ri

ultimate objective is to promote Indian


exports).
te

• Debt overhang refers to a debt burden so large that


an entity cannot take on additional debt to finance
Q.4) Ans: C
a

future projects.
• The burden is so large that all earnings pay off Exp:
m

existing debt rather than fund new investment EPCG Scheme was launched in the 1990s to facilitate
projects, making the potential for defaulting import of capital goods with the aim to enhance the
c

higher. production quality of goods and services, thereby,


s

• Debt overhangs can lead to underinvestment, increasing India’s international manufacturing


p

which stunts growth, making recovery even more competitiveness.


.u

difficult. • Statement 1 is correct: Under the scheme,


• Debt overhangs also apply to sovereign manufacturers can import capital goods for
w

governments. In these cases, the term refers to a pre-production, production and post-
situation in which the debt of a nation exceeds its production goods without attracting any
w

future capacity to repay it. This can occur from an customs duty on them. The exemption from
output gap or economic underemployment, paying the obligation of customs duty on the
w

repeatedly plugged by the creation of additional import of capital goods is subject to fulfilment
credit. A debt overhang can lead to stagnant
DPP 2023_DAY 188 18
https://upscmaterial.online/
Download From - https://upscmaterial.online/

Contact us :info@onlyias.com

OnlyIAS Nothing Else Visit :dpp.onlyias.in


Contact : +91-7007 931 912

of an export value equivalent to 6 times of duty ● Statement 3 is incorrect: In the last budget,
saved on the importation of such capital goods the deposit insurance cover was increased
within 6 years from the date of issuance of the from ₹1 lakh to ₹5 lakh per depositor per bank,
authorization. This would mean that the
fully protecting 98.1% of the total 252.6 crore
importer (being export-oriented) needs to
attract earnings in foreign currency which accounts or 50.9% of total assessable deposits
equals 600% of the customs duty saved in as at end-March 2021
domestic currency, within 6 years of availing
benefits of the Scheme. Q.7)
• Statement 2 is correct: EPCG is intended for Ans: C
promoting exports and the Indian Government Exp:
with the help of this scheme offers incentives
Under the present revised Economic Capital
and financial support to the exporters.
Framework or ECF from Bimal Jalan Committee the
amount of surplus that the RBI must transfer to the
Q.5) Ans: D Centre is determined by two factors as:
Exp: 1. Realised equity (essentially existing amount in
● Statement 1 is correct: India’s share in world CF) to be maintained within a range of 6.5% to
exports has increased from 0.6% in 1991 to 5.5% of the RBI’s balance sheet and transfer
1.7% in 2018. the excess amount from it.
● Statement 2 is correct: India’s services trade
2. Economic capital (essentially CGRA) to be kept
has been a major driver of its exports due to its
high growth and services trade surplus has in the range of 20-24.5% of the balance sheet
been financing almost 50% of India’s and the rest should be transferred to the

e
merchandise trade deficit. government.

n
● Statement 3 is correct: The Government has

li
set a merchandise export target of $400 billion Q.8) Ans: B
for the year 2021-22 anan d overall export

n
Exp:
target of $1 trillion exports by 2027.
● Statement 1 is incorrect: Targeted Long-Term
o
Repo Operations are Long term repo
l.
Q.6) Ans: A
Exp: operations (LTROs) conducted by the RBI to
a

● Statement 1 is correct: It registers a bank as ensure adequate liquidity at the longer period
ri

insured immediately and automatically on for specific sectors.


te

issuance of banking licence. ● Statement 2 is correct: Purpose of the TLTRO


● Statement 2 is correct: For all insured banks, is to ensure that there is enough liquidity in
a

Payment of deposit insurance premium to markets like the corporate market and their
m

DICGC is compulsory and DICGC pays the yields are not going up in the context of the
insured deposit amount to depositors in Covid set back. The TLTROs are exempted from
c

situations like - Liquidation (sale of all assets on regulatory requirement of the cash reserve
s

closing down of the bank), Reconstruction or ratio for the equivalent of incremental credit
p

any other arrangement under a scheme, or disbursed by banks as loans. Interest rate
under for banks will be floating rates linked to
.u

Merger or acquisition by another bank. It


covers all commercial banks, including the policy repo rate.
w

Payment Banks, Small Finance Banks, Regional ● Statement 3 is correct: In the context of the
Rural Banks, Foreign Bank branches in India, COVID-19 investors made large sell-offs in the
w

Local Area Banks and Co-operative Banks in all domestic equity, bond and forex markets. This
has led to higher yield or liquidity premia on
w

States and Union Territories.


instruments such as corporate bonds,

DPP 2023_DAY 188 19


https://upscmaterial.online/
Download From - https://upscmaterial.online/

Contact us :info@onlyias.com

OnlyIAS Nothing Else Visit :dpp.onlyias.in


Contact : +91-7007 931 912

commercial paper and debentures. To counter portfolio investors (FPIs), however, are
the high cost trend in corporate fund expected to face considerable operational
mobilisation, the RBI provided funds to banks challenges in adjusting to the new regime
because of the difference in time zones,
at concessional rate and for a higher tenure so
especially for the US and European investors
that the banks can invest these in corporate
bonds. Under TLTROs, the RBI will be Q.11) Ans: D
auctioning funds to commercial banks up to a Exp:
certain amount based on the repo rate. ● Statement 1 is incorrect: Floating rate funds
are those funds who buy bonds with changing
Q.9) Ans: C interest rates according to the changing rates
Exp: in the economy.
Option C is correct: In 2019, the Insolvency and ● Statement 2 is incorrect: Seen huge inflows in
Bankruptcy Board of India (IBBI) set up a working group
recent times due to expectations of rise in
for Tracking Outcomes of Insolvency and Bankruptcy interest rates, floating rates insulate investors
Code, 2016 (IBC), also known as GN Bajpai committee. from losses due to rate hikes. According to SEBI
The sole intention of the Insolvency and Bankruptcy rules, 65% of the corpus of floating rate funds
Code, 2016 is to provide a justified balance between must be invested in floating rate instruments.
and an interest of all the stakeholders of the company, A floating rate fund tries to replicate this kind
so that they enjoy the availability of credit the loss that
of payout for mutual fund investors by
a creditor might have to bear on account of default investing in bonds whose coupons (interest
• The objective behind Insolvency and payments) change according to overall interest

e
Bankruptcy Code, 2016 are listed below-
rates in the economy. This is meant to protect

n
1. To consolidate and amend the laws relating to
re-organization and insolvency resolution of investors from taking losses if rates are

li
corporate persons, partnership firms, and increased .

n
individuals.
2. To fix time periods for execution of the law in Q.12) Ans: A o
a time-bound settlement of insolvency (i.e. 180
l.
Exp:
days).
● Statement 1 is correct: NSE has launched NSE
a

3. To maximize the value of assets of interested


Prime as a new corporate governance
ri

persons.
4. To promote entrepreneurship initiative. NSE Prime is a framework that
te

5. To increase the availability of credit. prescribes higher standards of corporate


governance for listed companies than those
a

Q.10) Ans: B required by regulations.


m

Exp: ● Statement 2 is incorrect: It is open for all the


• Option B is correct: SEBI (Securities and NSE-listed companies to adopt voluntarily.
c

Exchange Board of India) has introduced an Listed companies that voluntarily choose to be
s

optional T+1 settlement cycle for the markets part of NSE Prime will need to comply with pre-
p

T+1 means that settlements will have to be defined norms on an ongoing basis, which will
cleared within one day of the actual
.u

be monitored by NSE.
transactions taking place. Increased market
liquidity and trading turnover with reduced
w

settlement risk and broker defaults. A switch to Q.13) Ans: B


the T+1 settlement cycle is expected to benefit Exp:
w

domestic investors by increasing market ● Statement 1 is correct: Anchor investors are


liquidity and trading turnover while reducing
w

institutional investors who are invited to


settlement risk and broker defaults. Foreign
subscribe the shares before the Initial Public
DPP 2023_DAY 188 20
https://upscmaterial.online/
Download From - https://upscmaterial.online/

Contact us :info@onlyias.com

OnlyIAS Nothing Else Visit :dpp.onlyias.in


Contact : +91-7007 931 912

Offers (IPOs) are made available to the public Investors who follow impact investing consider
as per Sebi regulations. An anchor investor in a a company's commitment to corporate social
public issue refers to a qualified institutional responsibility or the duty to positively serve
buyer (QIB) making an application for a value society as a whole.
of Rs 10 crores or more through the book- ● Statement 2 is correct: Socially responsible
building process. An anchor investor can (SRI) and environmental, social, and
attract investors to public offers before they governance (ESG) investing are two
hit the market to boost their confidence. An approaches to impact investing. Impact
anchor investor in an IPO is a qualified investing directs capital to those firms that
institutional buyer (QIB) like a foreign portfolio generate social or environmental benefits
investor or mutual fund or insurance company apart from profits. As an investor, you make
which invests before the IPO is made available investments with an intention to generate
to the public as per Sebi regulations. positive, measurable social and environmental
● Statement 2 is incorrect: Each investor is impact alongside a financial return. For
required to invest at least Rs 10 crore and the instance, investing in sectors such as
funds are subject to a 30-day lock-in period. sustainable agriculture, renewable energy,
Anchor investors are essentially brought in to conservation, microfinance, and affordable
enhance the confidence of investors and gauge and accessible basic services including housing,
the demand for the IPO in the market. healthcare, and education would be clubbed
under impact investing.
Extra Edge by OnlyIAS

e
Q.15) Ans: A

n
The anchor investor can’t sell his shares for at least 30 Exp:

li
days after the allotment. This rule ensures that ● Statement 1 is correct: Capital Adequacy

n
investors who want to flip shares on listing, do not use Ratio (CAR) is the ratio of a bank’s capital to
the ‘anchor’ route. Anchor investors can bid for shares its risk. It is also known as the Capital to Risk
o
anywhere within the price band declared by the (Weighted) Assets Ratio (CRAR). In other
l.
company. If the price discovered through the book- words, it is the ratio of a bank’s capital to its
a

building process is higher than the price at which risk-weighted assets and current liabilities. This
ri

shares were allotted to anchor investors, then these ratio is utilized to secure depositors and boost
te

investors have to bring in additional funds to make the efficiency and stability of financial systems
good the shortfall. But if the book built price is lower, all over the world. The CAR or the CRAR is
a

the excess amount is not refunded to them. computed by dividing the capital of the bank
m

with aggregated risk-weighted assets for credit


Q.14) Ans: B risk, operational risk, and market risk. This is
c

Exp: calculated by summing a bank’s tier 1 capital


s

Impact investing is an investment strategy that aims to and tier 2 capitals and dividing the total by its
generate specific beneficial social or environmental
p

total risk-weighted assets. That is: Tier 1 CAR


effects in addition to financial gains. Impact = (Eligible Tier 1 capital funds) = (Market Risk
.u

investments may take the form of numerous asset


RWA + Credit Risk RWA + Operational Risk
classes and may result in many specific outcomes.
w

RWA). Total CAR = (Eligible Total capital funds)


÷ (Credit Risk RWA + Market Risk RWA +
● Statement 1 is incorrect: Impact investing is a
w

general investment strategy that seeks to Operational Risk RWA)


● CAR Formula: CAR = (Tier 1 capital + Tier 2
w

generate financial returns while also creating a


capital)/risk weighted assets
positive social or environmental impact.
DPP 2023_DAY 188 21
https://upscmaterial.online/
Download From - https://upscmaterial.online/

Contact us :info@onlyias.com

OnlyIAS Nothing Else Visit :dpp.onlyias.in


Contact : +91-7007 931 912

● Statement 2 is not correct : Tier 1 capital: This ● Statement 2 is correct:The Scheme shall be
can absorb the losses without a bank being implemented on an ‘Area Approach basis’ i.e.,
required to stop trading. Also called core Defined Areas for each notified crop for
capital, this consists of ordinary share capital, widespread calamities with the assumption
equity capital, audited revenue reserves, and that all the insured farmers, in a Unit of
intangible assets. This is permanently available Insurance, to be defined as ‘Notified Area’ for
capital and readily available to absorb losses a crop, face similar risk exposures, incur to a
incurred by a bank without it having to cease large extent, identical cost of production per
operations. Tier 2 capital: This can absorb hectare, earn comparable farm income per
losses if the bank is winding-up and so gives hectare, and experience similar extent of crop
depositors a lesser measure of protection. This loss due to the operation of an insured peril, in
consists of unaudited reserves, unaudited the notified area.
retained earnings, and general loss reserves. ● Defined Area (i.e., unit area of insurance) is
This capital cushions losses if the bank is Village/Village Panchayat level by whatsoever
winding up and is used to absorb losses after a name these areas may be called for major
bank loses all its tier 1 capital. crops and for other crops it may be a unit of
size above the level of Village/Village
Q.16) Ans: C Panchayat.
Exp:
● Statement 1 is incorrect: PMFBY provides a ● Statement 3 is incorrect: The scheme was once
comprehensive insurance cover against failure mandatory for loanee farmers, but in 2020, the

e
of the crop thus helping in stabilising the Centre changed it to make it optional for all

n
income of the farmers. The Scheme covers all farmers. Earlier the rate of average premium

li
Food & Oilseeds crops and Annual subsidy including the difference between the

n
Commercial/Horticultural Crops for which past actuarial premium rate and the rate of the
yield data is available and for which requisite insurance premium payable by the farmer was
o
number of Crop Cutting Experiments (CCEs) shared by the state and center, further states
l.
are conducted under Crop Estimation Survey and UTs were free to extend additional
a

(GCES). The Scheme shall be implemented subsidies over and above the average subsidy
ri

through a multi-agency framework by from their budgets. The Centre decided in


te

selected insurance companies under the February 2020 to limit its premium subsidy to
overall guidance & control of the Department 30% for unirrigated areas and 25% for irrigated
a

of Agriculture, Cooperation & Farmers ones (from the existing unlimited). Previously,
m

Welfare (DAC&FW), Ministry of Agriculture & the central subsidy had no upper limit.
Farmers Welfare (MoA&FW), Government of
c

India (GOI) and the concerned State in co-


s

ordination with various other agencies; viz


p

Financial Institutions like Commercial Banks,


Co-operative Banks, Regional Rural Banks and
.u

their regulatory bodies, Government


w

Departments viz. Agriculture, Co-operation,


Horticulture, Statistics, Revenue, Q.17) Ans: B
w

Information/Science & Technology, Panchayati Exp:


Raj etc. ● Statement 1 is correct: According to the BIS
w

CPMI-MC Report (2018), CBDC does not alter

DPP 2023_DAY 188 22


https://upscmaterial.online/
Download From - https://upscmaterial.online/

Contact us :info@onlyias.com

OnlyIAS Nothing Else Visit :dpp.onlyias.in


Contact : +91-7007 931 912

the basic mechanics of monetary policy; from bank deposits, which could lead to a
rather, it has the potential to enable timely deposit outflow. This outflow of deposits (or
transmission of monetary policy. The banks’ durable liquidity) to CBDC could
implications of central bank digital currency motivate banks to compete for deposits, which
(CBDC) for monetary policy essentially in turn could increase deposit rates and thus
depends on the way it is designed and its also retail lending rates, despite no increase in
degree of usage. the policy rate. To avert this risk, the central
bank may have to proactively inject larger
● Statement 2 is correct: If the CBDC could durable liquidity to the banking system more
function like physical cash and fetch no interest regularly.
income, in normal times, economic agents
would prefer to keep their money in interest ● Statement 5 is correct: CBDCs shall be using
bearing bank deposits as opposed to CBDC. cryptography that will render the system safer
However, in the tail-risk event of economic as compared to the existing payment
instability or a system-wide bank run, CBDC infrastructure. However, the futuristic scenario
could be viewed as a safer substitute for bank of tackling quantum resistance also needs to
deposits. It is fully guaranteed by the central be factored in while designing the CBDCs.
bank with no risk of losing its face value and Quantum computers are machines whose
easily stored in large amounts. This easy switch processing power far outstrips even the most
to CBDC can potentially speed up a bank run. powerful supercomputers available today.
The consequent impairment to financial Quantum-resistant algorithms — also known

e
intermediation would directly weaken the as post-quantum, quantum-secure, and

n
efficacy of monetary policy. quantum-safe — are cryptographic algorithms

li
that can fend off attacks from quantum

n
● Statement 3 is incorrect: Potential demand for computers. Every CBDC will aim to be resistant
a CBDC is highly uncertain. It would be affected to quantum computing. Therefore, robustness
o
by its design and implementation framework of cryptographic techniques along with that of
l.
(Group of central banks (2021a and 2021b)). other encryption methods and ensuring
a

However, there are two main concerns: first quantum resistance shall be highly important
ri

that, in times of financial crisis, CBDC may to have a safe and secure CBDC technical
te

hypothetically result in faster bank runs; and infrastructure


second, financial disintermediation could lead
a

banks to rely on more expensive and less stable


m

sources of funding. However, such scenarios


can be addressed through appropriate limits
c

on CBDC holdings and transactions. Moreover,


s

if a run occurred, the central bank would be


p

more easily able to meet deposit withdrawal


requests with CBDC as opposed to cas
.u
w

● Statement 4 is correct: Interest bearing CBDC


could transmit monetary policy actions directly
w

to economic agents, increasing the efficiency


of monetary policy. Under this modality,
w

economic agents could also switch to CBDC

DPP 2023_DAY 188 23


https://upscmaterial.online/
Download From - https://upscmaterial.online/

Contact us :info@onlyias.com

OnlyIAS Nothing Else Visit :dpp.onlyias.in


Contact : +91-7007 931 912

requires that shareholding of the central


government in the specified insurers (the
above five companies) must be at least 51%.
The Bill removes this provision.

● Statement 3 is correct: The Act defines general


insurance business as fire, marine or
miscellaneous insurance business. It excludes
capital redemption and annuity from certain
businesses from the definition. Capital
redemption insurance involves payment of a
sum of money on a specific date by the insurer
after the beneficiary pays premiums
periodically. Under annuity certain insurance,
the insurer pays the beneficiary over a period
of time. The Bill removes this definition and
instead, refers to the definition provided by
Q.18) Ans: C the Insurance Act, 1938. Under the Insurance
Exp: Act capital redemption and annuity certain are
● Statement 1 is correct: The General Insurance included within general insurance business.
Business (Nationalisation) Amendment Bill,

e
2021 was introduced in Lok Sabha on July 30,

n
2021. The Bill seeks to amend the General Q.19) Ans: C
Insurance Business (Nationalisation) Act, 1972.

li
Exp:
The Act was enacted to nationalise all private • Statements 1 and 3 are incorrect: The notified

n
companies undertaking general insurance rules for Social Stock Exchange (SSE) to provide
business in India. The Bill seeks to provide for
o
social enterprises with an additional avenue to
l.
a greater private sector participation in the raise funds. Social enterprises (SEs) eligible to
a

public sector insurance companies regulated participate in the SSE will be entities, non-
profit organisations (NPOs) and for-profit
ri

under the Act.


social enterprises. Under the new rules, SSE
te

will be a separate segment of the existing stock


● Statement 2 is incorrect: The 1972 Act set up exchanges. NPOs are establishments that work
the General Insurance Corporation of India
a

for the welfare of society or the community


(GIC). The businesses of the companies and are set up as charitable associations. The
m

nationalised under the Act were restructured SSE aims to provide them with an alternative
in four subsidiary companies of GIC: (i) fund-raising avenue.The pandemic highlighted
c

the need for greater capital investments


National Insurance, (ii) New India Assurance,
s

toward voluntary organisations and


(iii) Oriental Insurance, and (iv) United India
p

enterprises working for social welfare. The SSE


Insurance. The Act was subsequently will help in this aspect by channelling greater
.u

amended in 2002 to transfer the control of capital to such organisations.Social enterprises


these four subsidiary companies from GIC to eligible to participate in the SSE will be entities
w

the central government, thereby making them - NPOs and for-profit social enterprises - having
social intent and impact as their primary
w

independent companies. Since 2000, GIC


goal.The idea of the Social Stock Exchange
exclusively undertakes reinsurance business.
w

(SSE) as a platform for listing social enterprise,


Government shareholding threshold: The Act voluntary and welfare organizations so that

DPP 2023_DAY 188 24


https://upscmaterial.online/
Download From - https://upscmaterial.online/

Contact us :info@onlyias.com

OnlyIAS Nothing Else Visit :dpp.onlyias.in


Contact : +91-7007 931 912

they can raise capital was mooted in the Union towards the company. Cash-strapped startups
Budget 2019-20.It may be listed on BSE or NSE. and business owners typically use sweat equity
• Social enterprise can be defined as a non-loss; to fund their companies. An equity share,
non-dividend paying company created and
normally known as ordinary share is a part
designed to address a social problem.It works
ownership where each member is a fractional
under the market regulator SEBI.The aim of the
initiative is to help social and voluntary owner and initiates the maximum
organisations which work for social causes to entrepreneurial liability related to a trading
raise capital as equity or debt or a unit of concern. These types of shareholders in any
mutual fund.Corporate foundations, political organization possess the right to vote.
or religious organisations or activities, ● Statement 2 is correct: Cash-strapped
professional or trade associations,
startups and business owners typically use
infrastructure and housing companies, except
affordable housing, will not be eligible to be sweat equity to fund their companies. Sweat
identified as a social enterprise.A Social Stock equity compensates for the shortage of cash.
Exchange may be helpful in rebuilding the The founders of start-up companies are often
livelihoods of people who are affected during disadvantaged by the lack of funds to finance
pandemics like COVID-19. their activities. However, they devote their
time to grow the company through effort and
• Statement 2 is correct: Social Stock Exchange
toil, which are rewarded back when the
will allow non-profit organisations to directly
list through issuance of bonds in the form of company becomes profitable. In real estate,
zero coupon or zero principal bonds. Zero- poor households often lack the funds to build
coupon bond is a debt security that does not their own homes but got a lot of free time on

e
pay interest but instead trades at a deep their hands. They can dedicate their time to

n
discount, drawing a profit at maturity, when building their own homes and those of their
the bond is redeemed for its full face value.This

li
neighbors. They also pay fewer mortgages
would help to access funds from donors,

n
than they would’ve paid if they purchased the
philanthropic foundations and Corporate
Social Responsibility (CSR) spenders as they houses. Also, sweat equity is as valuable as
o
will be encouraged to buy zero coupon bonds. cash equity. Often, large investors invest their
l.
money in small but growing companies with
a

the potential to become large companies in the


ri

Q.20) Ans: A future. Sweat equity allows companies to


Exp:
te

raise funds without raising debt levels.


● Statement 1 is Incorrect: The term sweat Startup companies often face challenges in
a

equity refers to a person or company's raising capital and obtaining too much debt
contribution toward a business venture or
m

may cripple the business. Sweat equity


other project. Sweat equity is a non-monetary provides them with a platform to get “free
c

contribution that the individuals or founders of money” by selling a portion of the company to
s

a company make towards the company. Sweat investors.


equity is generally not monetary and, in most
p

cases, comes in the form of physical labour, Q.21) Ans: C


.u

mental effort, and time. Sweat equity is Exp:


w

commonly found in real estate and the ● Statement 1 is correct: Domestic Systemically
construction industry, as well as in the Important Insurers (D-SIIs) refer to insurers of
w

corporate world—especially for startups.It is a such size, market importance and domestic and
non-monetary contribution that the global interconnectedness whose distress or
w

individuals or founders of a company make failure would cause a significant dislocation in

DPP 2023_DAY 188 25


https://upscmaterial.online/
Download From - https://upscmaterial.online/

Contact us :info@onlyias.com

OnlyIAS Nothing Else Visit :dpp.onlyias.in


Contact : +91-7007 931 912

the domestic financial system.The continued their attractiveness to investors.In green bonds,
functioning of D-SIIs is critical for the the money raised by the issuer are earmarked
uninterrupted availability of insurance services towards financing green projects like renewable
to the national economy.The parameters, as per energy, clean transportation etc.
the methodology, include: ● Statement 2 is incorrect: The World Bank is a
1. Size of opera tions in terms of total revenue, major issuer of green bonds. It has issued 164
including premium underwritten and the value such bonds since 2008, worth a combined $14.4
of assets under management. billion. In India, YES Bank was the first bank to
2. Global activities across more than one issue green bonds in 2015 for financing
jurisdiction. renewable and clean energy projects. The
● Statement 2 is incorrect: As Domestic Ghaziabad Municipal Corporation, a civic body in
Systemically Important Insurers (D-SIIs), they Uttar Pradesh, has become India’s first municipal
will also be subjected to enhanced regulatory corporation to successfully list the country’s first
supervision. IRDAI, just like the banking green municipal bonds on the Bombay Stock
regulator RBI did to identify such “too big to fail” Exchange (BSE) in April 2021.
banks and NBFCs, had endeavoured to identify ● Statement 3 is correct: Recently, Power Finance
such companies in the insurance business in the Corporation (PFC) launched its maiden €300
March of 2019, in the aftermath of the collapse million 7-year Euro Green Bond issuance.The
of IL&FS which triggered a massive liquidity crisis issuance saw strong participation from
in the financial markets. Presently, the State institutional investors across Asia and Europe
Bank of India (SBI), ICICI Bank, and HDFC Bank with participation from across 82 accounts and

e
have been identified as DSIBs in India. was oversubscribed 2.65 times.The

n
● Statement 3 is incorrect: Three insurers- Life overwhelming response to the issuance reflects

li
Insurance Corporation of India (LIC), General international investors’ confidence in PFC. This

n
Insurance Corporation of India (GIC) and New issuance also demonstrates a commitment to
India Assurance Co.- have been recognised as achieving India’s renewable energy goals. It is
o
Domestic Systemically Important Insurers (D- not only the first Euro bond issuance by PFC but
l.
SIIs) for 2020-21 by Insurance Regulator and also the first-ever Euro-denominated Green
a

Development Authority of India (IRDAI).The bond issuance from India. Moreover, it is the
ri

three public sector insurers shall raise the level first Euro issuance by an Indian non-banking
te

of corporate governance, identify all relevant finance corporation(NBFC) and the first Euro
risks and promote a sound risk management bond issuance from India since 2017. This bond
a

culture. D-SIIs shall be listed on an annual basis. issuance would help PFC in diversifying its
m

Size in terms of total revenue, including currency book as well as the investor base.
premium underwritten and the value of assets
c

under management are among the parameters


s

on which the insurers are identified. Q.23) Ans: C


p

Exp:
Q.22) Ans: B ● Statement 1 is correct: NDPS Act was enacted in
.u

Exp: 1985,the Act was amended three times in 1988,


w

● Statement 1 is correct: A green bond is a type of 2001 and 2014. It is the main legislation that
fixed-income instrument that is specifically deals with drugs and their trafficking. Various
w

earmarked to raise money for climate and provisions of the Act punish the production,
environmental projects. Green bonds may come manufacture, sale, possession, consumption,
w

with tax incentives and tax credits to enhance purchase, transport, and use of banned drugs

DPP 2023_DAY 188 26


https://upscmaterial.online/
Download From - https://upscmaterial.online/

Contact us :info@onlyias.com

OnlyIAS Nothing Else Visit :dpp.onlyias.in


Contact : +91-7007 931 912

except for medical and scientific purposes.Under Constitution has given varying degrees of
the Act, property acquired by a person from autonomy within the state legislature. Each
drug-related offences, who has been convicted autonomous district and regional council
under the Act can be seized, frozen and consists of not more than 30 members, of which
forfeited by the government. four are nominated by the governor and the rest
● Statement 2 is correct: Narcotics Control via elections.
Bureau is the nodal drug law enforcement and ● Statement 2 is correct: The ADCs are
intelligence agency of India responsible for empowered with civil and judicial powers, and
fighting drug trafficking and the abuse of illegal can constitute village courts within their
substance.It functions under Union Ministry of jurisdiction to hear the trial of cases involving
Home Affairs (MHA).All the offences under the the tribes. Governors of states that fall under the
NDPS Act are non-bailable. Also, no relief can be Sixth Schedule specify the jurisdiction of high
sought by the drug convicts by termination, courts for each of these cases.Along with ADCs,
remission, and commutation of sentences the Sixth Schedule also provides for separate
passed. The bail provision under NDPS requires Regional Councils for each area constituted as an
the court to have “reasonable grounds” to autonomous region.In all, there are 10 areas in
believe that the accused is not guilty and that he the Northeast that are registered as
is unlikely to commit another offence while on autonomous districts – three in Assam,
bail. Meghalaya and Mizoram and one in Tripura. All
● Statement 3 is incorrect: The NDPS Act defines of them remain in power for a term of five
cannabis (hemp) as a narcotic drug based on the years.Autonomous district councils have powers

e
parts of the plant that come under its purview. to levy taxes, fees and tolls on: building and land,

n
The Act, in its definition, excludes seeds and animals, vehicles, boats, entry of goods into the

li
leaves “when not accompanied by the tops”. area, roads, ferries, bridges, employment and

n
Bhang, which is made with the leaves of the income and general taxes for the maintenance of
plant, is not mentioned in the NDPS Act.Bhang schools and roads. o
is the edible preparation made from the leaves ● Statement 3 is incorrect: The power to organise
l.
of the cannabis plant, often incorporated into and reorganise the tribal areas as autonomous
a

drinks such as thandai and lassi, along with districts lie with the governor of the state. He can
ri

various foods. Bhang has been consumed in the also alter the name and boundary of such tribal
te

Indian subcontinent for centuries, and is areas. The governors of these(6th schedule)
frequently consumed during the festivals of Holi states are empowered to reorganize the
a

and Mahashivratri. boundaries of the tribal areas.In simpler terms,


m

she or he can choose to include or exclude any


Q.24) Ans: B area, increase or decrease the boundaries and
c

Exp: unite two or more autonomous districts into


s

● Statement 1 is incorrect: The Sixth Schedule one.They can also alter or change the names of
p

consists of provisions for the administration of autonomous regions without separate


tribal areas in Assam, Meghalaya, Tripura and legislation.
.u

Mizoram, according to Article 244 of the Indian


w

Constitution.Passed by the Constituent Q.25) Ans: A


Assembly in 1949, it seeks to safeguard the rights Exp:
w

of the tribal population through the formation of ● Statement 1 is incorrect: “contempt”, is


Autonomous District Councils (ADC).ADCs are understood as a set of legal provisions that
w

bodies representing a district to which the firewall courts, and safeguard and protect their

DPP 2023_DAY 188 27


https://upscmaterial.online/
Download From - https://upscmaterial.online/

Contact us :info@onlyias.com

OnlyIAS Nothing Else Visit :dpp.onlyias.in


Contact : +91-7007 931 912

ability to dispense justice. Article 129 of the by a private citizen as well — and in such a case,
Constitution conferred on the Supreme Court the consent of the Attorney General (or the
the power to punish contempt of itself. Article Advocate General, as the case may be,) is
215 conferred a corresponding power on the required.The motion or reference made for
High Courts. The Contempt of Courts Act, 1971, initiating the case will have to specify the
gives statutory backing to the idea.The prior contempt of which the person charged is alleged
consent of the Attorney General (AG) of India is to be guilty.The objective behind requiring the
not required to suo motu initiate the inherent consent of the A-G before taking cognizance of
contempt powers of the Supreme Court.The a complaint is to save the time of the court.
Contempt of Court Act of 1971 cannot limit this Judicial time is squandered if frivolous petitions
power of the court. The statute only provides the are made and the court is the first forum for
procedure in which such contempt is to be bringing them in. The A-G’s consent is meant to
initiated.The suo motu contempt powers of the be a safeguard against frivolous petitions, as it is
top court are drawn from Article 129 of the deemed that the A-G, as an officer of the court,
Constitution, which says the Supreme Court, as a will independently ascertain whether the
court of record, has the power to punish for complaint is indeed valid. In nutshell , The AG's
contempt of itself. consent is mandatory when a private citizen
● Statement 2 is correct: According to The wants to initiate a case of contempt of court
Contempt of Courts Act, 1971, contempt of against a person. However, when the court
court can either be civil contempt or criminal itself initiates a contempt of court case the AG's
contempt. consent is not required.

e
Civil contempt means “wilful disobedience to

n
any judgement, decree, direction, order, writ Q.26) Ans: D

li
or other process of a court, or wilful breach of Exp:

n
an undertaking given to a court”. ● Statement 1 is correct: Official language or
Criminal contempt, on the other hand, is languages of a State Subject to the provisions of
o
attracted by “the publication (whether by Article 346 and 347, the Legislature of a State
l.
words, spoken or written, or by signs, or by may by law adopt any one or more of the
a

visible representations, or otherwise) of any languages in use in the State or Hindi as the
ri

matter or the doing of any other act.It consists language or languages to be used for all or any
te

of three forms: of the official purposes of that State: Provided


(a) words, written or spoken, signs and actions that, until the Legislature of the State otherwise
a

that “scandalise” or “tend to scandalise” or provides by law, the English language shall
m

“lower” or “tends to lower” the authority of continue to be used for those official purposes
any court within the State for which it was being used
c

(b) prejudices or interferes with any judicial immediately before the commencement of this
s

proceeding and Constitution.


p

(c) interferes with or obstructs the ● Statement 2 is correct:Article 343 (1) of the
administration of justice. Constitution provides that Hindi in Devanagari
.u

● Statement 3 is incorrect: In the case of the script shall be the official language of the Union.
w

Supreme Court, the Attorney General or the Article 343 (2) also provides for continuing the
Solicitor General, and in the case of High Courts, use of English in official work of the Union for a
w

the Advocate General, may bring in a motion period of 15 years (i.e., up to January 25, 1965)
before the court for initiating a case of criminal from the date of commencement of the
w

contempt.However, the motion can be brought Constitution. Article 343 (3) empowered the

DPP 2023_DAY 188 28


https://upscmaterial.online/
Download From - https://upscmaterial.online/

Contact us :info@onlyias.com

OnlyIAS Nothing Else Visit :dpp.onlyias.in


Contact : +91-7007 931 912

Parliament to provide by law for continued use President’s order can only be made by
of English for official purposes even after January Parliament by law.
25, 1965. The Act also lays down under Section 3 ● Statement 3 is incorrect: As per the approved
(3) that both Hindi and English shall compulsorily Modalities(inclusion or exclusion), the proposals
be used for certain specified purpose such as made by State Governments/Union Territory
Resolutions, General Orders, Rules, Administrations are initially referred to the
Notifications, Administrative and other Reports, Registrar General of India(RGI) for comments.
Press Communiques; Administrative and other Proposals agreed to by the RGI are referred to
reports and official papers to be laid before a the National Commission for Scheduled Castes
House or the Houses of Parliament; contracts, (NCSC) for comments. Such proposals which
agreements, licences, permits, tender notices have been agreed to by both the RGI and the
and forms of tender, etc NCSC, as also approved by the Central
● Statement 3 is incorrect: The Constitution of Government, are incorporated in the Bill, which
India has not given any language a national is introduced in Parliament. As any amendment
status.Recently the Supreme Court dismissed a in the list of Scheduled Castes can be done only
PIL seeking directions to notify Sanskrit as the by an Act of Parliament in view of clause (2) of
national language. The Constitution has never Article 341 of the Constitution of India.
declared Hindi as the national language, rather,
in 1950, it has accorded Hindi in Devanagari Q.28) Ans: A
script to be the official language of the union Exp:
along with English under Article 343. ● Statement 1 is incorrect: Assisted suicide and

e
euthanasia both are practices under which a

n
Q.27) Ans: C person intentionally ends their life with active

li
Exp: assistance from others.Several European

n
● Statement 1 is incorrect: Article 342, which is nations, some states in Australia and Colombia
reproduced below, prescribes procedure to be in South America allow assisted suicide and
o
followed in the matter of specification of euthanasia under certain circumstances.Active
l.
scheduled tribes.The President may, with euthanasia, which is legal in only a few
a

respect to any State or Union territory, and countries, entails the use of substances to end
ri

where it is a state, after consultation with the the life of the patient.Passive euthanasia
te

Governor thereof by public notification, specify involves simply stopping lifesaving treatment
the tribes or tribal communities or parts of or or medical intervention with the consent of the
a

groups within tribes or tribal communities which patient or a family member or a close friend
m

shall, for the purposes of this constitution, is representing the patient. In other words Active
deemed to be scheduled tribes in relation to that euthanasia refers to the physician deliberate act,
c

state or Union Territory. Article 341 of the usually the administration of lethal drugs, to end
s

Constitution prescribes the procedure for an incurably or terminally ill patient's life.
p

recognising castes as “Scheduled Castes”. Passive euthanasia refers to withholding or


● Statement 2 is correct: The Constitution withdrawing treatment which is necessary for
.u

empowers the President to specify the maintaining life.


w

Scheduled Castes (SCs) in various states and ● Statement 2 is correct: In a landmark judgment,
union territories. Further, it permits Parliament the Supreme Court of India legalised passive
w

to modify this list of notified SCs. According to euthanasia in 2018, stating that it was a matter
Article 341 of the Constitution, any change to of ‘living will’. According to the judgement, an
w

the list of Scheduled Castes specified in the adult in his conscious mind is permitted to refuse

DPP 2023_DAY 188 29


https://upscmaterial.online/
Download From - https://upscmaterial.online/

Contact us :info@onlyias.com

OnlyIAS Nothing Else Visit :dpp.onlyias.in


Contact : +91-7007 931 912

medical treatment or voluntarily decide not to ➢ Every person who has been arrested
take medical treatment to embrace death in a would be produced before the nearest
natural way, under certain conditions. In the magistrate within 24 hours.
judgment, the court laid down a set of ➢ The custody of the detained person
guidelines for ‘living will’ and defined passive cannot be beyond the said period by
euthanasia and euthanasia as well.The court the authority of the magistrate.
specifically stated that the rights of a patient, in Article 22(3) says that the above safeguards are
such cases, would not fall out of the purview of not available to the following:
Article 21 (right to life and liberty) of the Indian
Constitution. ➢ If the person is at the time being an
enemy alien
Q.29) Ans: B ➢ If the person is arrested under certain
Exp: law made for the purpose of “Preventive
● Statement 1 is incorrect: Preventive Detention Detention”.
is the imprisonment of a person with the aim of
preventing him from committing further Q.30) Ans: B
offences or of maintaining public order.India is Exp:
one of the few countries in the world whose • The Constitution of India (under article 368 of
Constitution allows for preventive detention Part XX) has given the powers of making
during peacetime without safeguards that amendments to itself to the Indian Parliament.
elsewhere are understood to be basic It has mentioned that the Parliament can

e
change its procedures but cannot amend those
requirements for protecting fundamental
provisions which are the basis of the ‘basic

n
human rights. In short, preventive detention as structure’ of the Constitution.The Constitution

li
enshrined under Article 22 strikes a devastating can be amended in three ways: (1)

n
blow to personal liberties.A detainee under Amendment by a simple majority of the
preventive detention can have no right of Parliament (2) Amendment by a special
o
personal liberty guaranteed by Article 19 or majority of the Parliament. (3) Amendment by
l.
Article 21. a special majority of the Parliament and the
a

ratification of half of the state legislature.


● Statement 2 is correct: Article 22 of the Indian
• The Constitution (101th Amendment)
ri

Constitution provides protection against arrest Act,2017: Introduced the Goods and Services
te

and detention in certain cases. Article 22(1) and Tax in the country from 1 July 2017.
22(2) of the Indian constitution provides rights • The Constitution (102th Amendment)
a

of an Arrested Person in India. These are as Act,2018: It gave Constitutional status to the
National Commission for Backward Classes.
m

follows:
• The Constitution (103th Amendment)
➢ A person cannot be arrested and
Act,2019: It provided a maximum of 10%
c

detained without being informed why


Reservation for Economically Weaker Sections
s

he is being arrested. (EWSs).


➢ A person who is arrested cannot be
p

• The Constitution (104th Amendment)


.u

denied to be defended by a legal Act,2020: It extended the reservation of seats


practitioner of his choice. This means for SCs and STs in the Lok Sabha and states
w

that the arrested person has the right assemblies.


• The Constitution (105th Amendment) Act,
to hire a legal practitioner to defend
w

2021: It is the latest amendment in Indian


himself/ herself. Constitution 2021. The Bill seeks to restore the
w

power of State governments to identify OBCs


that are socially and educationally backward.
DPP 2023_DAY 188 30
https://upscmaterial.online/
Download From - https://upscmaterial.online/

Contact us :info@onlyias.com

OnlyIAS Nothing Else Visit :dpp.onlyias.in


Contact : +91-7007 931 912

Q.31) Ans: A ➢ The 2002 amendment inserted Section 33A


Exp: into the Act which provides for the right to
● Articles 324 to 329 of Part XV of the Indian information for people.
➢ After this, voters have the right to know the
Constitution provides for the country’s
antecedents of the candidates.
electoral system.The Constitution confers ➢ Contesting candidates are required to furnish
upon the Parliament the power to enact laws information about the prior conviction of
for all matters connected with elections to the offences or whether they are accused of any
Parliament and the State Legislature.The offence while filing their nominations.
government introduced the first RPA in 1950 ➢ The amendment also included provisions for
in order to regulate elections in the country. the declaration of assets and liabilities by the
candidates.
This Act makes provisions for:
➢ Lays down procedures for delimitation Q.32) Ans: C
of constituencies. Exp:
➢ Provides for the allocation of seats in • Statement 1 is correct: A cooperative is an
the House of the People and in the autonomous association of persons united
Legislative Assemblies and Legislative voluntarily to meet their common economic,
Councils of States. social and cultural needs and aspirations
➢ Lays procedure for the preparation of through a jointly-owned and democratically
controlled.The need for profitability is
electoral rolls and the manner of filling
balanced by the needs of the members and the
seats. wider interest of the community. The state
➢ Lays down the qualification of voters. legislature may make provisions for the

e
● Representation of People Act, 1951: incorporation, regulation and winding-up of

n
➢ This Act makes provisions for the co-operative societies based on the principles

li
of voluntary formation, democratic member
conduct of elections in India.
control, member economic participation and

n
➢ It also talks about corruption and autonomous functioning. The board shall
other illegal activities related to
o
consist of such a number of directors as may
l.
elections. be provided by the state legislature. But, the
a maximum number of directors of a co-
a

➢ The Act makes provisions for dispute


operative society shall not exceed twenty-
ri

redressal in matters connected to


one.The state legislature shall provide for the
elections.
te

reservation of one seat for the Scheduled


➢ It also talks about the qualification as Castes or the Scheduled Tribes and two seats
well as grounds for the for women on the board of every co-operative
a

disqualification of MPs and MLAs. society having members from such a category
m

● Representation of the People (Amendment) of persons. The term of office of elected


members of the board and its office bearers
Act, 1966:
c

shall be five years from the date of election.


➢ This Amendment abolished election tribunals.
s

• Statement 2 is incorrect: The Ninety-Seventh


Election petitions were now transferred to
p

Amendment Act of 2011 gave constitutional


High Courts.
status and protection to cooperative societies.
.u

➢ But disputes in connection with the


The Constitution (97th Amendment) Act, 2011
presidential and vice-presidential elections are
added a new Part IXB (Articles 243-ZH to 243-
w

heard directly by the supreme court of india.


ZT) right after Part IXA (Municipals) regarding
● Representation of the People (Amendment)
the cooperatives working in India.
w

Act, 2002: Reasons for this Amendment:


w

DPP 2023_DAY 188 31


https://upscmaterial.online/
Download From - https://upscmaterial.online/

Contact us :info@onlyias.com

OnlyIAS Nothing Else Visit :dpp.onlyias.in


Contact : +91-7007 931 912

➢ Cooperative sector has shown weaknesses in Sri Aurobindo launched Karmayogin, a weekly
safeguarding the interests of the members and English journal conceived as 'A Weekly Review
fulfilment of objectives. of National Religion, Literature, Science,
➢ For securing social and economic justice and
Philosophy.
equitable distribution of the fruits of
development. The illustration on the cover was that of Sri
➢ Inadequate professionalism in management in Krishna and Arjuna seated in their chariot on
many of the co-operative institutions has led to the battlefield of Kurukshetra, and the two
poor services and low productivity. mottos on the top were 'Remember me and
• Statement 3 is correct: The word fight' and 'Yoga is skill in works'.
“cooperatives” was added after “unions and
associations” in Article 19(1)(c) under Part ● Statement 3 is correct: He introduced the
III(Fundamental Rights) of the Constitution. world to the Indian philosophies of Vedanta
This enables all the citizens to form and Yoga. He preached ‘neo-Vedanta’, an
cooperatives by giving it the status of interpretation of Hinduism through a Western
fundamental right of citizens. A new Article lens, and believed in combining spirituality
43B was added in the Directive Principles of with material progress.He laid the greatest
State Policy (Part IV) regarding the “promotion emphasis on education for the regeneration of
of cooperative societies”. Co-operative our motherland. Advocated a man-making
Societies is a state subject under entry No.32 character-building education.
(7th schedule) of the of the Constitution of He is best known for his speech at the World
India Parliament of Religion in Chicago in 1893.

e
➢ Article 19 (1)(c): ‘Right to form cooperatives.’ • Associated Organisation: He was the chief

n
➢ Art. 43-B: “The state shall endeavour to disciple of the 19th-century mystic
promote voluntary formation, democratic

li
Ramakrishna Paramhansa and established the
control, autonomous functioning and Ramakrishna Mission in 1897.

n
professional management of cooperative • Ramakrishna Mission is an organisation which
societies”. o
works in the area of value-based education,
l.
culture, health, women's empowerment,
Q.33) Ans: C youth and tribal welfare and relief and
a

Exp: rehabilitation. In 1899, he established the


ri

● Statement 1 is incorrect: Swami Vivekananda Belur Math, which became his permanent
abode.
te

(January 12, 1863 - July 4, 1902) is one of the


most inspiring personalities of India.He was
Q.34) Ans: C
a

the principal disciple of Sri Ramakrishna


Exp:
m

Paramhansa. His efforts were acknowledged


● Statement 1 is correct: The G33 group has
even by the noted Indian leaders such as
advocated the creation of a "special products"
c

Mahatma Gandhi and Subhash Chandra Bose.


exemption, which would allow developing
s

Subhash Chandra Bose called him "the maker


countries to exempt certain products from
p

of modern India".
tariff reductions, and also a "special safeguard
.u

mechanism" which would permit tariff


● Statement 2 is incorrect:
increases in response to import surges.It has
w

Swami Vivekanand major literary works are :-


proposed special rules for developing countries
Sangeet Kalpatru, Karma Yoga, Raja Yoga,
w

at WTO negotiations on issues affecting food


Vedanta Philosophy, Lectures from Colombo
security and livelihood of farmers, like allowing
to Almora, Bartaman Bharat
w

My Master, Jnana Yoga.

DPP 2023_DAY 188 32


https://upscmaterial.online/
Download From - https://upscmaterial.online/

Contact us :info@onlyias.com

OnlyIAS Nothing Else Visit :dpp.onlyias.in


Contact : +91-7007 931 912

them to continue to restrict access to their 2. characterized by slow growth and an


agricultural markets. inherently weak balance of payments
● Statement 2 is incorrect: The G33 or ‘Friends of position.
Special Products (FoSP)’ in agriculture is a ● IMF is a specialised UN Agency conceived in
coalition of developing countries and least 1944 at United Nations Bretton Woods
developed countries pressing for flexibility for Conference. India is a founding member. The
developing countries to undertake limited EFF provides assistance in support of
market opening in agriculture. It has currently 48 comprehensive programs that include policies
member nations.It is a coalition of developing of the scope and character required to correct
countries, established prior to the 2003 Cancun structural imbalances over an extended
ministerial conference, that have coordinated period. Sri Lanka, in the midst of a crisis over
during the Doha Round of World Trade deterioration of the balance of payments
Organization negotiations, specifically in regard (BOP) position, has got the much-needed
to agriculture.Under this, both members India reprieve with the International Monetary Fund
and Pakistan are working closely at WTO despite (IMF) agreeing to provide $ 1.5 billion through
fractious bilateral relations. a three-year-long Extended Fund Facility
● Statement 3 is correct: The peace clause (EFF).Pakistan also received money from IMF
protects a developing country’s food under EFF.
procurement programmes against action from
WTO members in case subsidy ceilings are
breached. India was the first country to invoke

e
the peace clause for breaching the subsidy limit Q.36) Ans: C

n
for rice for the marketing year 2018-19. The limit Exp:
● Statement 1 is correct: As per IUCN,

li
is pegged at 10% of the value of food production
Transboundary Protected Areas(TPA) is an area

n
(called de minimis) in the case of India and other
developing countries. of land and/or sea that straddles one or more
o
borders between states, and whose constituent
l.
Q.35) Ans: A parts are dedicated to protection and
a

Exp: maintenance of biological diversity through legal


ri

● Extended Fund Facility (EFF) is an International or other effective means. Recently, the Union
Cabinet meeting chaired by the Prime Minister
te

Monetary Fund (IMF) lending facility to help


members with balance of payments problems approved the signing of a Memorandum of
Understanding (MoU) between India and Nepal
a

that need an adjustment period longer than


in the field of biodiversity conservation and
m

that provided for under a standby


arrangement.EFF support comes with many protection of transboundary protected areas.
● Statement 2 is incorrect: India has TPAs with
c

conditionalities that are meant to do away


Nepal and Bhutan.
s

with structural weaknesses in economy that


➢ Kanchenjunga Conservation Area: It is
p

have caused the BoP crisis.


The EFF was established to provide assistance located in the northeast corner of
.u

to countries: Nepal near the border with India and


Tibet.
w

1. experiencing serious payments


imbalances because of structural ➢ Terai Arc Landscape (TAL): It is spread
w

impediments; or across the Indian states of


Uttarakhand, Uttar Pradesh and Bihar
w

and low lying hills of Nepal.

DPP 2023_DAY 188 33


https://upscmaterial.online/
Download From - https://upscmaterial.online/

Contact us :info@onlyias.com

OnlyIAS Nothing Else Visit :dpp.onlyias.in


Contact : +91-7007 931 912

➢ Sacred Himalayan Landscape: Its 74% sanctuary is inundated by a number of rivers –


area falls in Nepal, 25% falls in Sikkim Brahmani, Baitarani, Dhamra, Pathsala.The
and remaining falls in Bhutan. park is famous for its green mangroves,
➢ Transboundary Manas Conservation migratory birds, turtles, estuarine crocodiles
Area: It is a transboundary landscape and countless creeks.It is said to house 70% of
across Easternn Himalayas, connecting the country’s estuarine or saltwater
Bhutan and Northeastern states of crocodiles, conservation of which was started
Assam and Arunachal Pradesh. way back in 1975.The crocodile conservation
● Statement 3 is correct: Transboundary project in Bhitarkanika was started in 1975.Its
Protected Areas (TPAs) ensures the long-term main objective was to protect the reptiles’
persistence of viable populations of species. It natural habitats and to rebuild the population
also helps to build greater ecological integrity by quickly through captive breeding as the
increasing the size of the area under survival rate of crocodile hatchlings in nature is
conservation management. It helps to maintain low because of predation.
and strengthen ecosystem resilience in the face
of climate change. It enhances cooperation in Q.38) Ans: D
transboundary and tourism planning. Exp:
Biodiversity conservation is the practice of ● All Statements are incorrect: The Martand Sun
protecting and preserving the wealth and variety Temple is a Hindu temple located near the city
of species, habitats, ecosystems, and genetic of Anantnag in the Kashmir Valley of Jammu
diversity on the planet, which is important for and Kashmir.It dates back to the eighth century

e
our health, wealth, food, fuel, and services we AD and was dedicated to Surya, the chief solar

n
depend on. It plays an integral role in supporting deity in Hinduism.It was built by the third ruler

li
many sectors of development. of the Karkota Dynasty, Lalitaditya

n
Muktapida.It is now in ruins, as it was
destroyed by the orders of Muslim ruler
o
Q.37) Ans: C Sikandar Shah Miri.
l.
Exp: ● From the ruins and related archaeological
a

● Bhitarkanika National Park is spread over a findings, it can be said it was an excellent
ri

vast area of 672 Kms in Orissa.It is a Ramsar specimen of Kashmiri architecture, which had
te

site and includes India's second-largest blended the Gandharan, Gupta and Chinese
mangrove forest. In the year 1988, it was forms of architecture.The temple appears in
a

designated as Bhitarkanika National Park.The the list of centrally protected monuments as


m

Gahirmatha Beach which forms the boundary Kartanda (Sun Temple)


of the sanctuary in the east is the largest
c

colony of the olive ridley sea turtles. Its ● Lalitaditya Muktapida


s

proximity to Bay of Bengal makes the soil of the Lalitaditya was born in the year of 699 AD as
p

area enriched with salts, the vegetation and the third son of Durlabhak-Pratapaditya of
kashmir. He was from the Nagvanshi Karkota
.u

the species of the sanctuary is comprised of


those which are mainly found in the tropical Kayastha Dynasty of Kashmir.
w

and subtropical inter tidal regions.It is home to Karkota Kayastha families were mainly serving
Saltwater Crocodile (Crocodylus porosus), in the army of the kings of Kashmir for decades.
w

White Crocodile, Indian python, King Cobra, They were known for their remarkable courage
on the battlefield.
w

black ibis, darters and many other species of


flora and fauna.The national park and wildlife

DPP 2023_DAY 188 34


https://upscmaterial.online/
Download From - https://upscmaterial.online/

Contact us :info@onlyias.com

OnlyIAS Nothing Else Visit :dpp.onlyias.in


Contact : +91-7007 931 912

The Kings of Kashmir had given them the title Exp:


Sakhasena for their immense contribution. ● Statement 1 is incorrect: Only the Great One-
Lalitaditya was a very liberal King, though he Horned Rhino is found in India.Also known as
was a strong follower of Hindu tradition, he Indian rhino, it is the largest of the rhino
respected all religions. He is said to be a very species.It is identified by a single black horn and
compassionate ruler who responded to a grey-brown hide with skin folds.They primarily
people’s voices. In the year 760 AD, the graze, with a diet consisting almost entirely of
Lalitaditya era came to an end by his sudden grasses as well as leaves, branches of shrubs and
death. trees, fruit, and aquatic plants. The Great one
horned rhino is commonly found in Nepal,
Bhutan, Pakistan and in Assam, India. It is
Q.39) Ans: B confined to the tall grasslands and forests in the
Exp: foothills of the Himalayas. The Indian Rhinoceros
● Statement 1 is incorrect: Lumpy skin disease is can run at speeds of up to 25 mph (40 km/h) for
caused by the lumpy skin disease virus (LSDV), short periods of time and is also an excellent
which belongs to the genus capripoxvirus, a swimmer. It has excellent senses of hearing and
part of the poxviridae family (smallpox and smell, but relatively poor eyesight.
monkeypox viruses are also a part of the same I. Javan rhinos are the most threatened of the
family).The LSDV shares antigenic similarities five rhino species, with only around 60
with the sheep pox virus (SPPV) and the goat pox individuals that live only in Ujung Kulon
virus (GTPV) or is similar in the immune response National Park in Java, Indonesia. Javan rhinos

e
to those viruses.It is not a zoonotic virus once lived throughout northeast India and

n
meaning the disease cannot spread to Southeast Asia. Vietnam’s last Javan rhino was

li
humans.The Lumpy Skin virus has infected over poached in 2010.

n
16 lakh cattle and killed nearly 75000 cattle in ● Statement 2 is incorrect: One-Horned Rhino
197 districts so far in India. comes under Schedule I of wildlife protection
o
● Statement 2 is correct: It is a contagious vector- act,1972 and vulnerable status on IUCN red list.
l.
borne disease spread by vectors like Also it comes under Appendix I (Threatened with
a

mosquitoes, some biting flies, and ticks and extinction and CITES prohibits international
ri

usually affects host animals like cows and water trade in specimens of these species except when
te

buffaloes. According to the United Nations Food the purpose of the import is not commercial, for
and Agriculture Organisation (FAO), infected instance for scientific research) of Convention on
a

animals shed the virus through oral and nasal International Trade in Endangered Species of
m

secretions which may contaminate common Wild Fauna and Flora (CITES).
feeding and water troughs. Thus, the disease can ● Statement 3 is correct: The five rhino range
c

either spread through direct contact with the nations (India, Bhutan, Nepal, Indonesia and
s

vectors or through contaminated fodder and Malaysia) have signed a declaration ‘The New
p

water. The disease was first observed in Zambia Delhi Declaration on Asian Rhinos 2019’ for the
in 1929, subsequently spreading to most African conservation and protection of the species.The
.u

countries extensively, followed by West Asia, Ministry of Environment Forest and Climate
w

Southeastern Europe, and Central Asia, and Change (MoEFCC) has begun a project to create
more recently spreading to South Asia and China DNA profiles of all rhinos in the country.The
w

in 2019. National Rhino Conservation Strategy was


launched in 2019 to conserve the greater one-
w

Q.40) Ans: C horned rhinoceros.Indian Rhino Vision 2020

DPP 2023_DAY 188 35


https://upscmaterial.online/
Download From - https://upscmaterial.online/

Contact us :info@onlyias.com

OnlyIAS Nothing Else Visit :dpp.onlyias.in


Contact : +91-7007 931 912

(IRV 2020) is an initiative led by the Forest ● Statement 3 is correct: Indian Council of
Department, Government of Assam, in Agricultural Research with its National
partnership with WWF India, International Rhino Agricultural Higher Education Project and Crop
Foundation, and several other organizations.The Science Division is organising Hackathon 3.0
goal of IRV2020 was to increase the rhino ‘’KRITAGYA” on promoting ‘speed breeding for
population in Assam to 3,000 by establishing crop improvement’.
populations in new areas. ICAR commenced NAHEP with the assistance
of the World Bank (WB) in November 2017.
The overall objective of NAHEP is to support
Q.41) Ans: D participating Agricultural Universities (AUs)
Exp: and ICAR in providing more relevant and higher
● Statement 1 is correct: Indian Council of quality education to the students.
Agricultural Research with its National
Agricultural Higher Education Project and Crop
Science Division is organising Hackathon 3.0 Q.42) Ans: D
‘’KRITAGYA” on promoting ‘speed breeding for Exp:
crop improvement’. This program will provide ● Statement 1 is incorrect: Also referred to as
an opportunity students/ faculties/ dead or doomed ice, zombie ice is one that is
entrepreneurs/ innovators and others to not accumulating fresh snow even while
showcase innovative approaches and continuing to be part of the parent ice sheet.It
technology solutions to promote innovation is because of the warming that has affected the

e
for crop improvement. climate already. The study focuses on an

n
The definition of KRITAGYA is: KRI for Krishi equipoise condition where snowfall from the

li
meaning Agriculture, TA for Taknik meaning northerly extremes of the Greenland ice cap

n
Technology and GYA for Gyan meaning melts down to the recharge edges of the snow
Knowledge. glaciers and hardens them. The study reveals
o
that over the past several decades there has
l.
● Statement 2 is correct: been less replenishment and more melting.
a

Eligibility: ● Statement 2 is incorrect: The recent study has


ri

1. Students, faculties and shown that melting of the Greenland ice sheet
te

innovators/entrepreneurs from any university will unavoidably raise the global sea levels by at
/ technical institution across the country can least 10.6 inches or 27 centimetres, no matter
a

apply and participate in the form of a group. what climate action the world decides to take
m

2. Participating students can collaborate with right now. This is because of ‘zombie ice’, which
local start-ups, students from technology is certain to melt away from the ice cap and
c

institutes. blend into the ocean which leads to increasing


s

3. The participating group shall consist of sea levels. Increasing sea levels will cause high
p

maximum 4 participants, with not more than tides, floods, and storms to occur more
one faculty and / or more than one innovator frequently. It will lead to threats to local
.u

or entrepreneur. economies and infrastructure. The low-lying


w

Participating students can collaborate with coastal regions will have to bear a harder hit.
local start-ups, students from technology
w

institutes and can win up-to INR 5 Lakhs.


w

Q.43) Ans: A

DPP 2023_DAY 188 36


https://upscmaterial.online/
Download From - https://upscmaterial.online/

Contact us :info@onlyias.com

OnlyIAS Nothing Else Visit :dpp.onlyias.in


Contact : +91-7007 931 912

Exp: saving to the nation on account of reduction in


● Statement 1 is correct : Advanced Chemistry import of crude-oil to a significant extent and
Cell (ACC) are the new generation of advanced increase the share of renewable at the national
storage technologies that can store electric grid level.
energy either as electrochemical or as
chemical energy and convert it back to electric
energy as and when required. Such battery Q.44) Ans: D
storages will cater not only to electric vehicles Exp:
but also to the consumer electronics industry ● Statement 1 is incorrect: Recently, India joined
and electricity grids. The consumer electronics, a multilateral strategic and command Exercise
electric vehicles, advanced electricity grids, Vostok - 2022 in Russia along with China
solar rooftop etc. which are major battery among other countries.
consuming sectors are expected to achieve It will involve troops from several ex-Soviet
robust growth in the coming years. Globally, nations, China, India, Laos, Mongolia,
manufacturers are investing in these new Nicaragua and Syria.The Indian Army was
generation technologies at commercial scale represented by the contingent of troops from
to fill the expected boom in battery demand 7/8 Gorkha Rifles.It is aimed at interaction and
through 2030. It is expected that the dominant coordination amongst other participating
battery technologies will control some of the military contingents and observers.Hence it
world's largest growth sectors. involves other countries as well. The Vostok
● Statement 2 is not correct : The Production 2022 exercise held at seven firing ranges in

e
Linked Incentive (PLI) scheme for Russia's Far East and the Sea of Japan and

n
manufacturers of Advanced Chemistry Cell involve more than 50,000 troops and over

li
(ACC) battery storage, to reduce imports is 5,000 weapons units, including 140 aircraft and

n
launched under Ministry of Heavy Industries 60 warships.The Indian Army contingent will
. The Government has approved the look forward to sharing practical aspects and
o
Production Linked Incentive (PLI) Scheme putting into practice the validated drills,
l.
'National Programme on Advanced Chemistry procedures and practice amalgamation of new
a

Cell (ACC) Battery Storage’ for achieving technology through discussions and tactical
ri

manufacturing capacity of Fifty (50) Giga Watt exercises.


te

Hour (GWh) of ACC for Enhancing India’s


Manufacturing Capabilities with a budgetary ● Statement 2 is incorrect: Special Forces of
a

outlay of ₹ 18,100 crore. Under the said Indian Army (Para Special Force) and
m

initiative the emphasis of the Government is Indonesian Army participate in Garuda Shakti
to achieve greater domestic value addition, exercise. The bilateral military exercise
c

while at the same time ensure that the between India and Indonesia had started off
s

levelized cost of battery manufacturing in from 2012


p

India is globally competitive. The Program


expects an investment that will boost domestic ● Statement 3 is incorrect: Exercise Hand in
.u

manufacturing & also facilitate battery storage Hand is a major military exercise between India
w

demand creation for both electric vehicles and and China.The aim of the exercise is to practice
stationary storage along with the development joint planning and conduct of counter terrorist
w

of a complete domestic supply chain & Foreign operations in semi urban terrain.
direct investment in the country. ACC PLI
w

scheme is expected to directly impact the Q.45) Ans: C

DPP 2023_DAY 188 37


https://upscmaterial.online/
Download From - https://upscmaterial.online/

Contact us :info@onlyias.com

OnlyIAS Nothing Else Visit :dpp.onlyias.in


Contact : +91-7007 931 912

Exp: conference on tourism.Dharamshala


● Statement 1 is incorrect: Recently, the Prime Declaration” affirms commitment toward
Minister of India commissioned India’s first developing “sustainable and responsible
indigenous aircraft carrier Indian Naval Ship tourism” and positions India as a “global leader
(INS) Vikrant in Kochi. in the tourism sector by 2047”.
Currently, India has two aircraft carriers INS The Centre unveiled an ambitious plan for the
Vikramaditya (from Russia) & INS Vikrant tourism sector in the National Conference that
(present one). The ship with a displacement of includes:
42,800 tonnes was designed by the Navy’s 1. Recovery of tourism to the pre-pandemic level
Warship Design Bureau (WDB) and built by by 2024,
Cochin Shipyard Limited (CSL), a public sector 2. $250 billion contribution to the GDP by 2030,
shipyard under the Ministry of Ports, Shipping 3. India will be a world leader in tourism by 2047.
and Waterway. Iyt has been built with state-of-
the-art automation features and is the largest The Dharamshala Declaration:
ship ever built-in the maritime history of India. It includes a long-term revenue goal of $1
trillion by 2047, when the country turns 100.
● Statement 2 is correct: The motto of the ship The Union Tourism and Culture Minister called
is Jayema Sam Yudhi Sprdhah which is taken on states to start work on “a war footing” by
from Rig Veda and is translated as “I defeat establishing tourism clubs. The proposal is to
those who fight against me” or We conquer work on making Yuva Tourism clubs at district
those who fight us'. and mandal levels.Private players can also be

e
involved in special cases, besides utilisation of

n
● Statement 3 is correct: the PM Gati Shakti initiative.

li
Operational capability:

n
1. The ship is capable of operating air wing Q.47) Ans: B
consisting of 30 aircraft comprising MiG-29K Exp: o
fighter jets, Kamov-31, MH-60R multi-role ● Statement 1 is correct : Classical computer
l.
helicopters, in addition to indigenously data is coded in either zeros or ones.
a

manufactured Advanced Light Helicopters Quantum information is superimposed in


ri

(ALH) and Light Combat Aircraft (LCA) (Navy). both zeros and ones simultaneously. A
te

2. The ship can accommodate an assortment of quantum network would allow the ultra secure
fixed-wing and rotary aircraft. transmission and exchange of quantum
a

The warship will also offer an “incomparable communications between distinct quantum
m

military instrument with its ability to project endpoints or devices over fiber optic cables.
Air Power over long distances, including Quantum devices will use their own “qubits” or
c

offensive, quantum bits — the equivalent of bits used by


s

Air Interdiction, Anti-Surface Warfare, ordinary computers but can be in a


p

defensive Counter-Air, Airborne Anti- superposition of both ‘0’ and ‘1.’ Information is
Submarine Warfare and Airborne Early stored in these qubits, which are encoded keys
.u

Warning. that are typically polarized photons. These


w

photons can travel very easily along fiber optic


Q.46) Ans: C cables.
w

Exp: ● Statement 2 is not correct : Quantum


● Recently, the ‘Dharamshala Declaration 2022’ networks apply uniquely quantum
w

was released at a three-day national phenomena, such as no-cloning,

DPP 2023_DAY 188 38


https://upscmaterial.online/
Download From - https://upscmaterial.online/

Contact us :info@onlyias.com

OnlyIAS Nothing Else Visit :dpp.onlyias.in


Contact : +91-7007 931 912

entanglement, and superposition. These environment surrounding Earth. These storms


phenomena are not available to ordinary result from variations in the solar wind that
internet networks. Photons exist in a produce major changes in the currents,
superposition of all their possible quantum plasmas, and fields in Earth’s magnetosphere
states and when they are measured, they are .The solar wind conditions that are effective for
forced to select one of these states.The two creating geomagnetic storms are sustained
most relevant aspects of quantum physics are (for several to many hours) periods of high-
the principles of superposition and speed solar wind, and most importantly, a
entanglement. southward directed solar wind magnetic field
● Superposition: Think of a qubit as an electron (opposite the direction of Earth’s field) at the
in a magnetic field. The electron’s spin may be dayside of the magnetosphere. This condition
either in alignment with the field, which is is effective for transferring energy from the
known as a spin-up state, or opposite to the solar wind into Earth’s magnetosphere.
field, which is known as a spin-down state. ● Statement 2 is not correct : The
According to quantum law, the particle enters magnetosphere shields our home planet from
a superposition of states, in which it behaves harmful solar and cosmic particle radiation, as
as if it were in both states simultaneously. Each well as erosion of the atmosphere by the solar
qubit utilized could take a superposition of wind – the constant flow of charged particles
both 0 and 1. streaming off the Sun.Not all solar flares reach
● Entanglement: Particles that have interacted Earth, but solar flares/storms, Solar Energetic
at some point retain a type of connection and Particles (SEPs), high-speed solar winds, and

e
can be entangled with each other in pairs, in a Coronal Mass Ejections (CMEs) that come close

n
process known as correlation. Knowing the can impact space weather in near-Earth space

li
spin state of one entangled particle “ up or and the upper atmosphere.

n
down “ allows one to know that the spin of its ● Statement 3 is not correct : Solar physicists
mate is in the opposite direction. Quantum and other scientists use computer models to
o
entanglement allows qubits that are separated predict solar storms and solar activities in
l.
by incredible distances to interact with each general. Current models are capable of
a

other instantaneously (not limited to the speed predicting a storm’s time of arrival and its
ri

of light). No matter how great the distance speed. But the storm’s structure or
te

between the correlated particles, they will orientation still cannot be predicted. Certain
remain entangled as long as they are isolated. orientations of the magnetic field can produce
a

a more intense response from the


m

Q.48) Ans: C magnetosphere, and trigger more intense


Exp: magnetic storms. With the increasing global
c

● Statement 1 is correct: Solar Storms occur dependence on satellites for almost every
s

during the release of magnetic energy activity, there is a need for better space
p

associated with sunspots (‘dark’ regions on the weather forecasts and more effective ways to
Sun that are cooler than the surrounding protect satellites.
.u

photosphere - the lowest layer of the solar


w

atmosphere), and can last for a few minutes or Q.49) Ans: B


hours. A geomagnetic storm is a major Exp:
w

disturbance of Earth's magnetosphere that ● Statement 1 is incorrect: Recently, an


occurs when there is a very efficient exchange unusually-shaped rainbow cloud appeared
w

of energy from the solar wind into the space over China. The cloud in question resembles a

DPP 2023_DAY 188 39


https://upscmaterial.online/
Download From - https://upscmaterial.online/

Contact us :info@onlyias.com

OnlyIAS Nothing Else Visit :dpp.onlyias.in


Contact : +91-7007 931 912

pileus cloud. A pileus cloud is usually formed


over a cumulus or cumulonimbus cloud and Q.50) Ans: B
not over a Nimbostratus cloud. Exp:
It is formed when the base cloud pushes a ● Statement 1 is correct: Solomon Islands is a
moist current of air upwards and the water country in the southwestern Pacific Ocean. It
vapor from the current condenses to consists of a double chain of volcanic islands
somewhat resemble wave-like crests or and coral atolls in Melanesia.
umbrellas. The Solomon Islands consist of six major and
A pileus cloud is transient in nature and lasts approximately 900 smaller volcanic islands,
barely for a few minutes, making it difficult, coral atolls and reefs.
and at the same time, exciting, to spot.
● Statement 2 is incorrect: Solomon Islands is
not part of the Kuril Island Group.
The archipelago nation consists of several large
volcanic islands to the south-east of Papua
New Guinea.
The Kuril Islands or Kuril Islands are a volcanic
archipelago currently administered as part of
Sakhalin Oblast in the Russian Far East. It
stretches approximately 1,300 km northeast
from Hokkaido in Japan to Kamchatka

e
● Statement 2 is correct: The phenomenon of Peninsula in Russia separating the Sea of

n
bright colours appearing on a cloud is called Okhotsk from the north Pacific Ocean.

li
cloud iridescence. Iridescent clouds, known as
"fire rainbows" or "rainbow clouds," occur

n
● Statement 3 is correct: The UK granted
when sunlight diffracts off water droplets in Solomon Islands internal self-government in
the atmosphere. And the recipe for these
o
1976, followed by independence on 7 July
l.
heavenly sights is actually pretty simple. Like 1978. At independence, Solomon Islands
a

common cloud-to-ground rainbows, iridescent joined the Commonwealth with Queen


ri

clouds usually accompany thunderstorms. Elizabeth II as its Head of State, represented by


iridescence or Irisation is an optical
te

a Governor-General.
phenomenon that mostly occurs in wave-like The Commonwealth of Nations, simply
clouds, including pileus Altocumulus
a

referred to as the Commonwealth is a political


lenticularis.
m

association of 56 member states, the vast


Iridescence in clouds means the appearance of majority of which are former territories of the
c

colours on clouds, which can either be in the British Empire.The chief institutions of the
form of parallel bands like in a rainbow or
s

organisation are the Commonwealth


mingled in patches.
p

Secretariat, which focuses on


In ancient Greek mythology, Iris is the goddess intergovernmental aspects, and the
.u

of the rainbow. “Irisation”, the phenomenon of Commonwealth Foundation, which focuses on


rainbow-like colours in clouds, is derived from
w

non-governmental relations amongst member


her name. The iridescence of clouds is a states.
w

photometer – an optical phenomenon


produced by the reflection, refraction,
w

diffraction, or interference of sunlight.

DPP 2023_DAY 188 40


https://upscmaterial.online/
Download From - https://upscmaterial.online/

Contact us :info@onlyias.com

OnlyIAS Nothing Else Visit :dpp.onlyias.in


Contact : +91-7007 931 912

due to the processes of evaporation and


transpiration.
3. Potential evapotranspiration is the maximum
attainable or achievable evapotranspiration
for a given crop due to evaporation and
transpiration.
Applications:
Impacts of drought on agriculture, especially in
the tropics where defined wet and dry seasons
are part of the climate regime.
Both winter and summer cropping seasons can
be assessed using this method.
Q.51) Ans: B
Exp: Q.52) Ans: B
● Statement 1 is incorrect: The Aridity Anomaly Exp:
index monitors agricultural drought, a ● Statement 1 is incorrect: Cervavac, India’s first
situation when rainfall and soil moisture are indigenously developed quadrivalent human
inadequate to support healthy crop growth till papillomavirus (qHPV) vaccine for the
maturity, causing crop stress. An anomaly from prevention of cervical cancer has been
the normal value signifies a water shortage in developed. A quadrivalent vaccine is a vaccine
these districts that could directly impact that works by stimulating an immune response

e
agricultural activity. against four different antigens, such as four

n
It is Developed by the India Meteorological different viruses or other

li
Department (IMD). microorganisms.Cervavac was developed by the

n
Pune-based Serum Institute of India in
● Statement 2 is correct: coordination with the Government of India’s
o
Characteristics: Department of Biotechnology (DBT).Unlike
l.
1. A real-time drought index in which water Covid vaccines, booster shots may not be
a

balance is considered. required for the cervical cancer vaccine. Until


ri

2. The Aridity Index (AI) is computed for weekly now, the HPV vaccines available in India were
te

or two-week periods. produced by foreign manufacturers at an


3. For each period, the actual aridity for the approximate cost of Rs 2,000 to Rs 3,500 per
a

period is compared to the normal aridity for dose. Cervavac is likely to be significantly
that period.
m

cheaper, slated to cost approximately Rs. 200 to


4. Negative values indicate a surplus of moisture 400.
c

while positive values indicate moisture stress ● Statement 2 is correct: Cervical cancer is a
s

prevalent sexually transmitted infection.It’s a


● Statement 3 is incorrect:
p

type of cancer that occurs in the cells of the


Parameters: cervix, the lower part of the uterus that connects
.u

1. Actual evapotranspiration and calculated to the vagina. It is mostly caused by long-term


w

potential evapotranspiration, which require infection with particular forms of HPV. It is the
temperature, wind and solar radiation values. second most prevalent cancer form and the
w

2. Actual evapotranspiration is the quantity of second leading cause of cancer death in women
water that is actually removed from a surface of reproductive age (15-44).
w

DPP 2023_DAY 188 41


https://upscmaterial.online/
Download From - https://upscmaterial.online/

Contact us :info@onlyias.com

OnlyIAS Nothing Else Visit :dpp.onlyias.in


Contact : +91-7007 931 912

● Statement 3 is correct: Cervavac has a significant liquids facilitates throttling and control over the
potential to eliminate cervical cancer and it flow rate of LOX.
would be helpful if it will be included in national
HPV vaccination efforts and offered at a lower Q.54) Ans: A
cost than existing vaccinations. According to the Exp:
World Health Organisation (WHO), India ● Statement 1 is not correct: NBFCs, also known
accounts for about a fifth of the global burden as nonbank financial institutions, are entities
of cervical cancer, with 1.23 lakh cases and that provide certain-bank like and financial
around 67,000 deaths per year. Almost all services but do not hold a banking license and
cervical cancer cases are linked to certain strains excludes entities with agriculture activity,
of human papillomavirus (HPV), a common virus industrial activity, purchase or sale of any
that is transmitted through sexual contact. goods (other than securities) as principal
Despite being largely preventable, cervical business or providing any service and
cancer is the fourth most common cancer among sale/purchase/construction of immovable
women globally, according to the WHO. property. Principal Business is the financial
activity which consists of more than 50% of
Q.53) Ans: D company assets and over 50% if it’s gross
Exp: income.Regulation: NBFCs are regulated by RBI
● Statement 1 is incorrect: The Indian Space although some NBFCs are regulated by SEBI,
Research Organisation (ISRO) has successfully IRDAI, National Housing Bank etc. to avoid dual
tested a hybrid propulsion system.The 30 kN regulation.

e
hybrid motor tested at ISRO Propulsion Complex ● Statement 2 is correct : Systemically

n
(IPRC) at Mahendragiri in Tamil Nadu.The test important NBFCs: NBFCs whose asset size is of

li
was supported by ISRO's Liquid Propulsion ₹ 500 crore or more are considered as

n
Systems Centre (LPSC).It uses a solid fuel and systemically important NBFCs. Example. Power
liquid oxidiser.The motor used Hydroxyl- Finance Corporation Limited (PFCL), Rural
o
terminated polybutadiene (HTPB) as fuel and Electrification Corporation Limited (RECL) etc.
l.
liquid oxygen (LOX) as the oxidiser. ● Statement 3 is not correct : NBFCs differ from
a

● Statement 2 is correct: While both HTPB and banks in following ways : NBFCs can’t accept
ri

LOX are green, the cryogenic LOX is safer to demand deposits; NBFCs are not part of the
te

handle.The hybrid system is more efficient, payment and settlement system and can’t
”greener” and safer to handle and paves the issue cheques drawn on itself; and DICGC
a

way for new propulsion technologies for future facility is not available to depositors of NBFCs.
m

missions.The technology demonstration paves Deposit Insurance and Credit Guarantee


the way for hybrid propulsion-based sounding Corporation (DICGC) is a specialised division of
c

rockets and an exciting platform for vertical Reserve Bank of India which is under the
s

landing experiments for spent-stage recovery. jurisdiction of the Ministry of Finance,


p

● Statement 3 is incorrect: Conventional HTPB- Government of India. It was established on 15


based solid propellant motors used in rockets July 1978 under the Deposit Insurance and
.u

use ammonium perchlorate as oxidiser. In rocket Credit Guarantee Corporation Act, 1961 for the
w

engines, oxidisers supply the oxygen needed for purpose of providing insurance of deposits
combustion.Unlike conventional solid motors, and guaranteeing of credit facilities. DICGC
w

the hybrid technology permits restarting and insures all bank deposits, such as saving, fixed,
throttling capabilities on the motor. The use of current, recurring deposit for up to the limit of
w

Rs. 500,000 of each depositor in a bank. The

DPP 2023_DAY 188 42


https://upscmaterial.online/
Download From - https://upscmaterial.online/

Contact us :info@onlyias.com

OnlyIAS Nothing Else Visit :dpp.onlyias.in


Contact : +91-7007 931 912

limit was increased from 1 lakh to 5 lakh in corrective measures, to restore the financial
2020. The deposit insurance facility of Deposit health of a bank.
Insurance and Credit Guarantee Corporation ● PCA Framework for NBFCs shall come into
is not available to depositors of NBFCs, unlike effect from 1st October 2022, based on the
in case of banks. financial position of NBFCs on or after March
31, 2022. PCA Framework for NBFCs shall apply
Q.55) Ans: B to All Deposit Taking NBFCs [Excluding
Exp: Government Companies] All Non-Deposit
● Statement 1 is not correct : Prompt Corrective Taking NBFCs in Middle, Upper and Top Layers
Action is a qualitative tool introduced by the . PCA excludes: NBFCs not accepting/not
Reserve Bank of India under which direct intending to accept public funds; Government
action is taken on weak banks to ensure the Companies; Primary Dealers and Housing
financial health of a bank. PCA is a framework Finance Companies
under which financial institutions, usually ● Statement 3 is correct : Stressed banks may be
banks and now NBFCs as well, with weak prohibited from expansion of credit portfolio
financial metrics are put under watch by the and asked to restrict outsourcing activities,
RBI. It was first introduced in 2002 as a going by the Reserve Bank of India’s revised
structured early-intervention mechanism for prompt corrective action (PCA) framework. A
banks that become under capitalised due to bank prohibited from expansion of
poor asset quality, or vulnerable due to loss of credit/investment portfolios under PCA will,
profitability. It was last reviewed in 2017 based however, be allowed to invest in government

e
on the recommendations of the Financial securities/other high-quality liquid

n
Stability and Development Council and the investments. As per the extant framework, the

li
Financial Sector Legislative Reforms RBI can ask a bank under PCA to only

n
Commission. restrict/reduce credit expansion for borrowers
● Statement 2 is not correct : RBI has also issued below certain rating grades, reduce exposure
o
Prompt Corrective Action (PCA) Framework to unsecured borrowers, among others. But it
l.
for Non-Banking Financial Companies (NBFCs) does not prohibit expansion of
a

to tighten regulation over weaker NBFCs. It credit/investment portfolios.


ri

will cover All Deposit Taking NBFCs [NBFCs-D,


te

Excluding Government Companies]; All Non- Q.56) Ans: A


Deposit Taking NBFCs in Middle, Upper and Exp:
a

Top Layers (NBFCs-ND) including Core ● Statement 1 is not correct : Suggested by


m

Investment Companies (CICs) [Excluding - (i) Nachiket Mor committee in 2013, it is a bank
NBFCs not accepting/not intending to accept registered under Companies Act, 2013 and
c

public funds; (ii) Government Companies, (iii) licensed under Banking Regulation Act, 1949.
s

Primary Dealers and (iv) Housing Finance Minimum paid-up equity capital of the
p

Companies].Previously, the PCA framework payments bank is ₹100 crores and it is required
was applicable only to commercial banks and to maintain a minimum capital adequacy ratio
.u

not extended to co-operative banks, non- of 15%. Promoters of the payments bank
w

banking financial companies (NBFCs)and FMIs. should hold at least 40% of its paid-up equity
Now urban cooperative banks (UCBs) will also capital for the first 5 years. Promoter
w

fall under its ambit. PCA helps RBI monitor key /promoter group can have Joint Venture with
performance indicators of banks, and taking an existing SCB.
w

DPP 2023_DAY 188 43


https://upscmaterial.online/
Download From - https://upscmaterial.online/

Contact us :info@onlyias.com

OnlyIAS Nothing Else Visit :dpp.onlyias.in


Contact : +91-7007 931 912

● Statement 2 is not correct : They operate on a to DICGC is compulsory and DICGC pays the
smaller scale with no credit risk (can’t advance insured deposit amount to depositors in
loans or issue credit cards). It can accept situations like-Liquidation (sale of all assets on
demand deposits (up to Rs 1 lakh), offer closing down of the bank), Reconstruction or
remittance services, mobile purchases and any other arrangement under a scheme, or
other banking services payments/transfers/ Merger or acquisition by another bank.
They maintain Cash Reserve Ratio (CRR) with ● Statement 2 is correct : Key Features of DICGC
RBI and invest in eligible government (Amendment) Act, 2021 : Introduction of
securities/ treasury bills under Statutory Liquid interim payments from DICGC to these
Ratio (SLR). They can access interbank depositors with 90 days (max) timeline.
uncollateralized call money market and Provisions for penal interest in case of delay
collateralized repo and CBLO market for to ensure timely repayment from banks to
temporary liquidity management. DICGC. Removal of earlier ceiling of 15 paise
● Statement 3 is correct : Recently, SEBI allowed (0.15%) on premium and DICGC to notify the
the payment banks to act as investment ceiling on premium with prior RBI approval.
bankers, i.e. to work in raising capital for Empower DICGC’s Board to defer due
corporations, governments, or other entities repayments from insured banks after
like Goldman Sachs, Morgan Stanley.In a insurance pay out on its own terms.
notification dated July 30, the regulator ● Statement 3 is not correct : It covers all
amended the Bankers to an Issue rules, commercial banks, including Payment Banks,
thereby permitting such other banking Small Finance Banks, Regional Rural Banks,

e
company, as may be specified by the Sebi, from Foreign Bank branches in India, Local Area

n
time to time, to carry out the activities of Banks and Co-operative Banks in all States

li
Bankers to an Issue (BTI), in addition to the and Union Territories. In the last budget, the

n
scheduled banks. Bankers to an issue mean a deposit insurance cover was increased from ₹1
scheduled bank or such other banking lakh to ₹5 lakh per depositor per bank, fully
o
company as may be specified by Sebi carrying protecting 98.1% of the total 252.6 crore
l.
activities, including acceptance of application accounts or 50.9% of total assessable deposits
a

money, acceptance of allotment or call money, as at end-March 2021. Internationally, fully


ri

refund of application money and payment of protected accounts are around 80% and 20-
te

dividend or interest warrants. 30% of total accessible deposits. Bank-group


wise, the percentage of insured deposits vis-à-
a

Q.57) Ans: C vis total deposits is 84% for RRBs, 70% for
m

Exp: cooperative banks, 59% for SBI, 55% for PSBs,


● Statement 1 is not correct : DICGC is a wholly 40% for private sector banks and 9% for foreign
c

owned subsidiary of the RBI created under banks.


s

the DICGC Act, 1961 with Deputy Governor of


p

RBI as its ex-officio Chairman. Governed by Q.58) Ans: A


the provisions of the DICGC Act of 1961 and Exp:
.u

DICGC General Regulations of 1961, DICGC is ● Statement 1 is not correct : D-SIBs : The
w

liable to pay the insured deposit amount to system of D-SIBs was adopted in the
depositors of an insured bank. It registers a aftermath of the 2008 financial crisis where
w

bank as insured immediately and automatically the collapse of many systemically important
on issuance of banking license. For all insured banks across various regions further fueled
w

banks, Payment of deposit insurance premium the financial downturn. D-SIBs are important

DPP 2023_DAY 188 44


https://upscmaterial.online/
Download From - https://upscmaterial.online/

Contact us :info@onlyias.com

OnlyIAS Nothing Else Visit :dpp.onlyias.in


Contact : +91-7007 931 912

for the country’s economy. In events of ● Statement 3 is not correct : The funds
distress, the government supports such banks transferred through NEFT are processed in 12
and if such a bank fails, it would lead to batches between 8:00 AM to 6:30 PM on
disruption of the country’s overall economy. weekdays and between 8:00 AM and 1:00 PM
RBI finalizes such banks after considering on Saturdays. It is not available on Sundays and
factors like size, complexity, lack of bank holidays. The settlement of funds
substitutability and interconnectedness of happens on a half-hourly basis in NEFT . In
the banks, state report. D-SIBs are financial RTGS , The settlement of funds happens in
institutions that are large enough where they real time. This means they are processed as
cannot be allowed to fall. RBI places D-SIBs in they are made.
appropriate buckets depending upon their
Systemic Importance Scores (SISs). The central Q.60) Ans: A
bank’s current update on D-SIBs is based on Exp:
the data collected from banks as of March 31, ● Statement 1 is correct :Money multiplier is a
2021. term in monetary economics that is a
● Statement 2 is correct : Due to their economic phenomenon of creating money in the
and national importance, the banks need to economy in the form of credit creation, which
maintain a higher share of risk-weighted is based on the fractional reserve banking
assets as tier-I equity. SBI, since it is placed in system. Money multiplier is also known as the
bucket three of D-SIBs, has to maintain monetary multiplier. It is the maximum limit to
Additional Common Equity Tier 1 (CET1) at which money supply can be affected by

e
0.60 percent of its Risk-Weighted Assets bringing about changes in the amount of

n
(RWAs). money deposits. The money multiplier effect is

li
seen in commercial banks as they accept

n
Q.59) Ans: B deposits, and after keeping a certain amount
Exp: as a reserve, they distribute the money as
o
● National Electronic Funds Transfer (NEFT) is loans for injecting liquidity in the economy.
l.
an online system used for transferring small to ● Statement 2 is not correct : There is an inverse
a

large amounts of money from one financial relationship between the Money Multiplier
ri

entity to another within India. Real-Time Gross and the legal reserve ratio. The amount of
te

Settlement (RTGS) is a funds transfer system money that should be kept by commercial
where funds of high quantity are transferred banks in their reserve for withdrawal
a

from one bank to another in ‘real-time’ and on purposes by the customers is referred to as
m

a gross basis. The similarity between them is the reserve ratio, required reserve ratio, or
that they are both commonly used methods of cash reserve ratio. Mathematically, money
c

transferring money online. multiplier formula can be represented as


s

● Statement 1 is correct : The NEFT mode is used follows: Money multiplier = 1/r ; Where r =
p

when the transactions are of smaller values. Required reserve ratio or cash reserve ratio. It
RTGS is used in high-value transactions. means that if the reserve ratio is higher, then
.u

● Statement 2 is not correct : The National the money multiplier will be lower and the
w

Electronic Funds Transfer method does not banks need to keep more reserves. As a result,
have a minimum transfer limit ceiling. The they will not be able to lend more money to
w

minimum amount needed to be transferred individuals and businesses. Similarly, a lower


has to be of Rs. 2 Lakhs and above for RTGS reserve ratio results in a higher money
w

multiplier that allows a lesser amount of

DPP 2023_DAY 188 45


https://upscmaterial.online/
Download From - https://upscmaterial.online/

Contact us :info@onlyias.com

OnlyIAS Nothing Else Visit :dpp.onlyias.in


Contact : +91-7007 931 912

money to be kept as a reserve and more supply. The interbank deposits, which a
lending opportunities to the public. commercial bank holds in other commercial
banks, are not to be regarded as part of money
Q.61) Ans: C supply.
Exp:
● Statement 1 is correct : The total stock of Q.62) Ans: A
money in circulation among the public at a Exp:
particular point of time is called money ● Statement 1 is correct : The Payments Bank is
supply. It needs to be noted that total stock of proposed to be registered as a public limited
money is different from total supply of money. company under the Companies Act of 2013,
Supply of money is only that part of total stock and licenced under Section 22 of the Banking
of money which is held by the public at a Regulation Act of 1949. It will be governed by
particular point of time.The circulating money the provisions of the Banking Regulation Act of
involves the currency, printed notes, money in 1949, the Reserve Bank of India Act of 1934,
the deposit accounts and in the form of other the Foreign Exchange Management Act of
liquid assets. 1999, the Payment and Settlement Systems
● Statement 2 is correct : Money Supply is Act of 2007.
measured and expressed using different ● Statement 2 is correct : A Payments Bank is
monetary aggregates like M1, M2, M3, M4 etc. similar like banks, but they are operating on a
Terms like Narrow Money and Broad Money smaller scale without involving any credit risk.
are also used to denote money supplyRBI They can operate both current as well as saving

e
publishes figures for four alternative measures accounts and can provide all other services like

n
of money supply, viz. M1, M2, M3 and M4; M1 ATM cards, fund transfers, bill payments,

li
(Narrow Money) =Currency with the public + recharges, net banking among others to the

n
Deposit money of the public (Demand deposits account holders. They can’t advance loans or
with the banking system + ‘Other’ deposits issue credit cards, and cannot accept deposits
o
with the RBI); M2 = M1 + Savings deposits with from the Non-Resident Indians (NRIs).
l.
Post Office savings banks; M3 = M1 + Net time ● Statement 3 is not correct : They are not
a

deposits of commercial banks ; M4 = M3 + authorised to get involved in lending activities.


ri

Total deposits with Post Office savings It is mandatory for the payments bank to
te

organisations (excluding National Savings maintain Cash Reserve Ratio (CRR) with the
Certificates). M1 and M2 are known as narrow Reserve Bank of India. The payments bank has
a

money. M3 and M4 are known as broad to invest at least 75% of its demand deposit
m

money. balance in the Government security or


● Statement 3 is not correct : These gradations Treasury Bills.
c

are in decreasing order of liquidity. M1 is


s

most liquid and easiest for transactions Q.63) Ans: A


p

whereas M4 is least liquid of all. M3 is the Exp:


most commonly used measure of money ● Statements 1 and 2 are correct : Small finance
.u

supply. It is also known as aggregate banks (SFBs) are a type of niche banks in India.
w

monetary resources.CU is currency (notes plus They can be promoted either by individuals,
coins) held by the public and DD is net demand corporate, trusts or societies. They are
w

deposits held by commercial banks. The word governed by the provisions of Reserve Bank of
‘net’ implies that only deposits of the public India Act, 1934, Banking Regulation Act, 1949
w

held by the banks are to be included in money and other relevant statutes. They are

DPP 2023_DAY 188 46


https://upscmaterial.online/
Download From - https://upscmaterial.online/

Contact us :info@onlyias.com

OnlyIAS Nothing Else Visit :dpp.onlyias.in


Contact : +91-7007 931 912

established as public limited companies in the ● Statement 2 is not correct : Foreign Direct
private sector under the Companies Act, Investments (FDI) is hard for the investors to
2013. Banks with a SFB license can provide depart from the country due to the
basic banking service of acceptance of considerable cost involved. Foreign Portfolio
deposits and lending. The Objective of setting- Investments (FPI) Investors can quickly depart
up an SFB is to provide financial inclusion to from the country as they invest in stocks and
sections of the economy not being served by bonds, which are liquid.
other banks, such as small business units, small ● Statement 3 is correct : Foreign institutional
and marginal farmers, micro and small investors (FIIs) are companies based outside
industries and unorganized sector entities. of India that make investment proposals in
Existing non-banking financial companies the country. They have a significant impact on
(NBFC), microfinance institutions (MFI) and a country's economy. Hedge funds, mutual
local area banks (LAB) can apply to become funds, pension funds, insurance bonds, high-
small finance banks. value debentures, and investment banks are
● Statement 3 is not correct : The banks will not examples of institutional investors. The
be restricted to any region. 75% of its net Securities and Exchange Board of India has
credits should be in priority sector lending and around 1450 FIIs registered (SEBI).
50% of the loans in its portfolio must be ₹25
lakh. The firms must have a capital of at least Q.65) Ans: A
₹200 crore. The promoters should have 10 Exp:
years’ experience in banking and finance. ● Statement 1 is not correct : InvITs are like

e
Foreign shareholding will be allowed in these mutual funds that pool money from investors.

n
banks as per the rules for FDI in private banks They are regulated by Securities and Exchange

li
in India. Board of India (SEBI) . InvITs could be set up

n
for sectors defined under the infrastructure as
per RBI guidelines. InvITs own and operate
o
Q.64) Ans: B operational infrastructure assets like
l.
Exp: highways, roads, pipelines, warehouses, power
a

● Statement 1 is not correct : Foreign direct plants, Typically, vehicles like InvITs are
ri

investment (FDI) is an investment made by a designed to pool money (small sums) from
te

company or an individual in one country into several investors to be invested in income-


business interests located in another country. generating assets. They offer regular income
a

FDI is an important driver of economic growth. (via dividends) and long-term capital
m

Foreign direct investment (FDI) includes appreciation.


investments in high-yielding assets such as ● Statement 2 is correct : An InvIT is established
c

the plant and machinery of a business. When as a trust settled by the sponsor under the
s

investors from some other country buy Indian Trusts Act, 1882 and the trust deed
p

securities or some other financial assets, this registered in India under the Registration Act,
is referred to as a foreign portfolio investment 1908. Besides, a Certificate of Registration as
.u

(FPI). Since portfolio investments can be easily an InvITs needs to be obtained from SEBI.
w

sold off and are therefore perceived as a short- Distributions by InvITs are based on Net
term effort to make money instead of a long- distributable cash flows (NDCF), unlike
w

term investment in the economy, this sort of companies where dividends are based on
investment is sometimes viewed less positively profits. At present, there are 15 InvITs
w

than direct investment.

DPP 2023_DAY 188 47


https://upscmaterial.online/
Download From - https://upscmaterial.online/

Contact us :info@onlyias.com

OnlyIAS Nothing Else Visit :dpp.onlyias.in


Contact : +91-7007 931 912

registered with SEBI, and seven are listed on cost efficient, ETFs offer a diversified
the stock exchanges. investment portfolio to investors.
● Statement 3 is not correct : An InvIT must ● Option C is not correct : Alternative mutual
invest at least 80% of its total assets in funds (sometimes called alt funds or liquid alts)
completed infrastructure projects capable of are publicly offered, SEC-registered mutual
generating income. The remainder of assets funds that hold non-traditional investments or
up to a limit of 20% held by the InvIT can be use complex investment and trading
invested in under-construction infrastructure strategies. Investors considering alt funds
projects and various SEBI-approved Equity, should be aware of their unique characteristics
Debt, and Money Market instruments. and risks.
● Option D is not correct : A hedge fund is a
Q.66) Ans: B limited partnership of private investors whose
Exp: money is managed by professional fund
● Option A is not correct : A Mutual fund is a managers who use a wide range of strategies,
pool of investments by different investors in including leveraging or trading of non-
securities such as debt, equity or both. In traditional assets, to earn above-average
simple words, it collects funds from investors investment returns. Hedge fund investment is
like individuals and institutions and invests often considered a risky alternative investment
them in bonds, stocks or other short-term choice and usually requires a high minimum
investment plans. Generally, a fund manager investment or net worth, often targeting
oversees this fund and charges a minimal wealthy clients.

e
amount called expense ratio for the same.

n
Besides, the Securities and Exchange Board of Q.67) Ans: C

li
India or SEBI regulates and moderates the Exp:

n
mutual funds industry like security markets. ● Option C is correct: Bond Yield : Yield is the
● Option B is correct : Exchange Traded Fund : amount of return an investor will receive on a
o
An Exchange-Traded Fund (ETF) is a basket of bond. The principal and interest on the bond
l.
securities that trade on an exchange, just like a will be returned to the investor if they retain
a

stock. ETF reflects the composition of an Index, the bond until it matures. The investors do not
ri

like BSE Sensex. Its trading value is based on need to keep a bond until it matures, though.
te

the Net Asset Value (NAV) of the underlying As an alternative, investors may sell them to
stocks (such as shares) that it represents. ETF other investors for a greater or lower price.
a

share prices fluctuate all day as it is bought and Bond yields and prices typically move in the
m

sold. This is different from mutual funds that opposite directions. This is due to the fact that
only trade once a day after the market closes. a bond's yield to maturity decreases as its price
c

An ETF can own hundreds or thousands of rises. The normal yield curve is a yield curve in
s

stocks across various industries, or it could be which short-term debt instruments have a
p

isolated to one particular industry or sector. lower yield than long-term debt instruments of
Bond ETFs are a type of ETFs which may include the same credit quality.
.u

government bonds, corporate bonds, and state ● Yield inversion : When the yield on a bond
w

and local bonds—called municipal bonds. A with a longer term becomes lower than the
bond is an instrument that represents a loan yield on a bond with a shorter term, this is
w

made by an investor to a borrower (typically known as yield inversion. A recession is


corporate or governmental). Besides being generally predicted by a yield inversion. An
w

inverted yield curve indicates that investors

DPP 2023_DAY 188 48


https://upscmaterial.online/
Download From - https://upscmaterial.online/

Contact us :info@onlyias.com

OnlyIAS Nothing Else Visit :dpp.onlyias.in


Contact : +91-7007 931 912

anticipate a steep decline in future growth; in Q.69) Ans: C


other words, they anticipate that the demand Exp:
for money will decline significantly from its ● Statement 1 is not correct : SGBs are
current level, which will result in lower yields. government securities denominated in grams
Inverted yield curves are unusual since longer- of gold. They are substitutes for holding
term debt should carry greater risk and higher physical gold. Investors have to pay the issue
interest rates, so when they occur there are price in cash and the bonds will be redeemed
implications for consumers and investors alike. in cash on maturity. The Bond is issued by
An inverted yield curve is one of the most Reserve Bank on behalf of the Government of
reliable leading indicators of an impending India. The quantity of gold for which the
recession. investor pays is protected, since he receives
the ongoing market price at the time of
Q.68) Ans: C redemption/ premature redemption. The SGB
Exp: offers a superior alternative to holding gold in
● Money Market : A random course of financial physical form. The risks and costs of storage
institutions, bill brokers, money dealers, banks, are eliminated. Investors are assured of the
etc., wherein dealing on short-term financial market value of gold at the time of maturity
tools are being settled is referred to as Money and periodical interest. SGB is free from issues
Market. These markets are also called like making charges and purity in the case of
wholesale markets. Money Market is a gold in jewellery form. The bonds are held in
disorganised market, so the dealing is done off the books of the RBI or in demat form

e
the public exchange market, i.e. Over The eliminating risk of loss of scrip etc.

n
Counter (OTC), within two bodies by using ● Statement 2 is not correct : The Bonds are

li
email, fax, online and phones, etc . Money issued in denominations of one gram of gold

n
Market Since they are extremely liquid in and in multiples thereof. Minimum
nature, the money market recovery period is investment in the Bond shall be one gram
o
restricted to one year. A few examples of with a maximum limit of subscription of 4 kg
l.
Money Market are: Trade Credit ; Commercial for individuals, 4 kg for Hindu Undivided Family
a

Paper ; Certificate of Deposit ; Treasury Bills (HUF) and 20 kg for trusts and similar entities
ri

● Capital Market : A kind of financial market notified by the government from time to time
te

where the company or government securities per fiscal year (April – March).
are generated and patronised with the ● Statement 3 is correct: These securities are
a

intention of establishing long-term finance to eligible to be used as collateral for loans from
m

coincide with the capital necessary is called banks, financial Institutions and Non-Banking
Capital Market. The Capital Market instrument Financial Companies (NBFC). The Loan to
c

involves both the auction market and dealer Value ratio will be the same as applicable to
s

market. It is classified into two sections: ordinary gold loans prescribed by RBI from
p

Primary Market and Secondary Market. The time to time. Granting loans against SGBs
capital market circulates the capital in the would be subject to the decision of the
.u

economy among the user and the suppliers of bank/financing agency, and cannot be inferred
w

money. The maturity period is more than one as a matter of right.


year or sometimes it is incurable (no maturity).
w

Capital Market Examples : Stocks ; Bonds ; Q.70) Ans: C


Debentures ; Shares, Asset Secularisation. Exp:
w

DPP 2023_DAY 188 49


https://upscmaterial.online/
Download From - https://upscmaterial.online/

Contact us :info@onlyias.com

OnlyIAS Nothing Else Visit :dpp.onlyias.in


Contact : +91-7007 931 912

● Option A is not correct : AT-1 bonds are a type diversification of options hence acting as a
of unsecured, perpetual bonds that banks issue substitute for a bank guarantee.
to shore up their core capital base to meet the ● Option D is not correct : Masala Bonds : They
Basel-III norms.There are two routes through are bonds issued outside India by an Indian
which these bonds can be acquired: Initial entity or corporate. These bonds are issued in
private placement offers of AT-1 bonds by Indian currency rather than local currency.
banks seeking to raise money ; Secondary Indian corporates usually issue Masala Bonds
market buys of already-traded AT-1 bonds. AT- to raise funds from foreign investors. As it is
1 bonds are like any other bonds issued by pegged into Indian currency, if the rupee rates
banks and companies, but pay a slightly higher fall, investors bear the risk. The first Masala
rate of interest compared to other bonds. bond was issued in 2014 by IFC for the
These bonds are also listed and traded on the infrastructure projects in India.
exchanges. So, if an AT-1 bondholder needs
money, he can sell it in the secondary market. Q.71) Ans: C
● Option B is not correct : A social impact bond Exp:
(SIB) is a contract with the public sector or ● Option C is correct : Certificate of Deposit :
governing authority, whereby it pays for better Certificate of Deposit (CDs) are short-term
social outcomes in certain areas and passes on instruments issued by Commercial Banks and
part of the savings achieved to investors. A Special Financial Institutions (SFIs), which are
social impact bond is not a bond, per se, since freely transferable from one party to another.
repayment and return on investment (ROI) are The maturity period of CDs ranges from 91

e
contingent upon the achievement of desired days to one year. These can be issued to

n
social outcomes. If the objectives are not individuals, cooperatives and companies.A

li
achieved, investors receive neither a return Certificate of Deposit in India can be issued by:

n
nor repayment of principal All scheduled commercial banks excluding
● Option C is correct : surety bond : A surety Regional Rural Banks (RRBs) and Local Area
o
bond is defined as a three-party agreement Banks (LABs) Select All India Financial
l.
that legally binds together a principal who Institutions permitted by RBI. A commercial
a

needs the bond, an obligee who requires the bank can issue Certificate of Deposit as per its
ri

bond, and a surety company that sells the own requirements. A financial institution can
te

bond. The bond guarantees the principal will issue Certificate of Deposit within a limit
act by certain laws. If the principal fails to prescribed by RBI. A thumb rule for FI is that CD
a

perform in this manner, the bond will cover the together with other instruments, viz. term
m

resulting damages or losses. The obligee, money, term deposits, commercial papers and
usually a government entity, requires the inter-corporate deposits should not exceed
c

principal, typically a business owner or 100 per cent of its net-owned funds, as per the
s

contractor, to obtain a surety bond as a latest audited balance sheet. Certificate of


p

guarantee against future work performance.A Deposit can be issued to individuals,


surety bond is provided by the insurance corporations, companies, trusts, funds,
.u

company on behalf of the contractor to the associations etc. The No resident Indians are
w

entity which is awarding the project. These also eligible for CDs provided they don’t
bonds are mainly aimed at infrastructure repatriate the funds.
w

development to reduce indirect costs for


suppliers and contractors. This helps in the Q.72) Ans: B
w

Exp:

DPP 2023_DAY 188 50


https://upscmaterial.online/
Download From - https://upscmaterial.online/

Contact us :info@onlyias.com

OnlyIAS Nothing Else Visit :dpp.onlyias.in


Contact : +91-7007 931 912

● Statement 1 is correct :Collective Investment risk provided by hedging also typically results
Scheme is a scheme under which the in a reduction in potential profits. Hedging
contributions made by the investors are requires one to pay money for the protection
pooled and the units of the schemes are listed it provides, known as the premium.Hedging
on a stock exchange. There is a two-tier also involves the use of multiple concurrent
structure consisting of the Collective bets in the opposite directions with the aim of
Investment Management Company (CIMC) and limiting the risk of serious investment losses. It
trustees. The CIMC floats and manages a CIS, is not a pursuit of risk-free trades, but it helps
whereas the trustees are the guardian of the to reduce the known risks that can occur while
funds. A system offered by a company under trading. Forward contracts, options contracts,
which the contributions made by the investors swaps and derivatives are used by the traders
are pooled and used with the goal of receiving so that they can purchase opposite positions in
profits, income, produce or property is known a financial market. If the trader can bet both
as a Collective Investment Scheme (CIS). against the upward and downward
● Statement 2 is not correct : Sebi tightens movements in the price of a particular item,
norms on collective investment schemes, they can ensure a certain amount of gain or
eases securities transmission process . loss on the trade. This method of betting is
According to a Sebi statement, The regulator particularly important in areas like business
has increased the net-worth criteria and management, financial markets and
enhanced the track record requirement of gambling. Like the other risk or reward-
promoters for running a collective investment related trades, the hedging can result in lower

e
management company (CIMC). Sebi has also returns for a trader, but it can also offer

n
restricted a CIMC and its group shareholders to significant protection against the risk of a

li
10 per cent shareholding or representation on downside.

n
the board of another CIMC to avoid conflict of
interest. It also said that a CIMC and its Q.74) Ans: B o
designated employees should have a Exp:
l.
mandatory investment in the CIS to align their ● Statement 1 is not correct : Treasury bills or T-
a

interests with that of the CIS. The regulator bills, which are money market instruments,
ri

fixed a minimum number of investors and are short term debt instruments issued by the
te

subscription amount at the CIS Government of India and are presently issued
level.Minimum 20 investors and a in three tenors, namely, 91 day, 182 day and
a

subscription amount of at least ₹20 crores is 364 day. T-bills are the instruments for raising
m

mandated by SEBI for each CIS. It also funds by the government for the short-term up
rationalised the fee and expenses to be to a year.
c

charged to the scheme and reduced the ● Statement 2 is not correct : Treasury bills are
s

timeline for the offer period of scheme and zero coupon securities and pay no interest,
p

refund of money to investors. Instead, they are issued at a discount and


redeemed at the face value at maturity. For
.u

Q.73) Ans: B example, a 91 day Treasury bill of ₹100/- (face


w

Exp: value) may be issued at say ₹ 98.20, that is, at


● Option B is correct : Hedging is a risk a discount of say, ₹1.80 and would be
w

management strategy employed to offset redeemed at the face value of ₹100/-. The
losses in investments by taking an opposite return to the investors is the difference
w

position in a related asset. The reduction in between the maturity value or the face value

DPP 2023_DAY 188 51


https://upscmaterial.online/
Download From - https://upscmaterial.online/

Contact us :info@onlyias.com

OnlyIAS Nothing Else Visit :dpp.onlyias.in


Contact : +91-7007 931 912

(that is ₹100) and the issue price (for legislation to regulate chit funds (e.g., The
calculation of yield on Treasury Bills. These bills Kerala Chitties Act, 1975, and The Tamil Nadu
are issued by the Central government only and Chit Funds Act, 1961 etc).
interest rate is determined by the market ● Statement 2 is not correct : At present chit
forces. It is issued at a discount to original value funds are governed by Chit Funds Act of 1962,
and the buyer gets the original value upon RBI Act of 1934, and SEBI Act of 1992 etc.
maturity. For example, a Rs 100 treasury bill Under the Chit Fund Act of 1962, businesses
can be availed of at Rs 90, but the buyer is paid can be registered and regulated only by the
Rs 100 on the maturity date. They are issued respective State Governments. Regulator of
via auctions conducted by the Reserve Bank of chit funds is the Registrar of Chits appointed
India (RBI) at regular intervals. Individuals, by respective state governments under
trusts, institutions and banks can purchase T- Section 61 of Chit Funds Act.The Chit Funds
Bills. The returns on T-bills are influenced by Act of 1982 governs the management,
the liquidity as well. When there is a liquidity operation, and regulation of chit funds in India.
crisis then the yield is higher on the T-bills. It is They are governed by central legislation, Chit
considered a very safe financial instrument for Funds Act of 1982 , but their administration is
zero-risk weighting attached to it. the responsibility of state governments.
● Statement 3 is correct : Banks give treasury ● Statement 3 is correct : Ponzi schemes are
bills to the RBI to get money under repo. investment operations that pay returns to old
Similarly, they can also keep it to fulfil their investors from the money garnered from new
Statutory Liquid Ratio (SLR) requirements. investors. Ponzi Schemes: Ponzi schemes are

e
Treasury bills, or T-bills, have a maximum investment operations that pay returns to

n
maturity period of 364 days and are thus called existing investors using funds raised from new

li
money-market instruments. Currently they are participants (Pyramid schemes). Despite the

n
issued in three maturities: 91-day, 182-day and existence of strict restrictions against chit fund
364-day (an earlier 14-day bill was also issued). frauds, many of these funds operate Ponzi
o
schemes and steal a lot of people's money.
l.
Q.75) Ans: A
a

Exp: Q.76) Ans: B


ri

● Statement 1 is not correct : A chit fund is a Exp:


te

form of savings plan in which a certain ● Statement 1 is correct: A depository receipt is


number of people donate money in a negotiable (transferable) financial
a

installments over a set period of time. instrument or certificate issued by a bank to


m

Depending on the form of the chit fund, each reflect the security of a foreign company's
subscriber is entitled to a reward sum publicly traded securities that are exchanged
c

determined by lot, auction, or tender. on a local stock exchange, usually in the form
s

Typically, the prize is equal to the total amount of equity, shares, or other securities.American
p

of contributions minus a discount, which is depositary receipts, European depository


then given as a dividend to subscribers.Chit receipts, global depository receipts, Indian
.u

funds are part of the Concurrent List of the depository receipts are among the types of
w

Indian Constitution, according to the Supreme depository receipts. When a foreign firm
Court, which classifies them as contracts. As a wants to list its publicly traded shares or
w

result, both the centre and the states can enact securities on an international stock exchange,
chit fund legislation. States such as Tamil Nadu, it must first issue a depository receipt. Before
w

Andhra Pradesh, and Kerala have passed listing its stocks for sale, a corporation must

DPP 2023_DAY 188 52


https://upscmaterial.online/
Download From - https://upscmaterial.online/

Contact us :info@onlyias.com

OnlyIAS Nothing Else Visit :dpp.onlyias.in


Contact : +91-7007 931 912

adhere to the stock exchange's special rules. A ● Statement 2 is not correct : If the fund raised
company's stock must be transferred to a is in the foreign currency, the borrowers must
brokerage house in its home nation. After pay a significant amount of servicing debt at
receiving the shares, the brokerage house sells the time of depreciating the domestic
them by issuing depository receipts through a currency. The issuers don’t have to deal with
custodian connected to the foreign exchange. the transfer risk because of the fluctuations in
The depository receipts can be exchanged over the currency. Maturity period: If Masala Bond
the counter in the open market. is raised to USD 50 million eq in Rs, the financial
● Statement 2 is not correct : An Indian year should be 3 years, and the minimum
Depository Receipt (IDR) is a financial maturity period should be 5 years.
instrument that allows a foreign company to ● Statement 3 is correct : Masala Bond is issued
raise funds in India . In an IDR, a foreign by private and government businesses as well.
company issues shares to an Indian International Finance Corporation (IFC) issued
Depository, which then issues depository the first Masala bond to build infrastructure in
receipts (IDR) to Indian investors. An Overseas India in 2014, therefore also known as IFC
Custodian would hold the real shares Masala Bonds.
underlying the IDRs and authorize the Indian
Depository to issue the IDRs. IDRs are Q.78) Ans: B
derivative instruments in this sense because Exp:
their value is derived from the underlying ● Statement 1 is not correct : The Reserve Bank
shares. of India’s Financial Inclusion Index (FI-Index)

e
has improved to 56.4 in March 2022 vis-à-vis

n
Q.77) Ans: B 53.9 in March 2021, with growth witnessed

li
Exp: across all the sub-indices. The index is

n
● Statement 1 is not correct : Masala bonds are published annually in July. The RBI developed
rupee-denominated bonds issued in India by the composite financial inclusion index to
o
a corporate and Indian entity, i.e. these are capture the extent of financial inclusion
l.
the funds raised in Indian currency from the across the country by including details of
a

overseas market. They can be termed as the banking, investments, insurance, postal as
ri

debt products which will provide an aid in the well as the pension sector.The Index is
te

raising of funds in local currency from investors responsive to ease of access, availability and
who are foreign or are international. These usage of services, and quality of services. The
a

bonds can be issued by both the private index captures information on various aspects
m

entities or business and also the government- of financial inclusion in a single value ranging
based entities. These bonds are available to between 0 and 100, where 0 represents
c

investors who are foreign investors or are from complete financial exclusion and 100 indicates
s

outside India who want to invest in Indian full financial inclusion. The index has been
p

assets. These bonds, which are members of the constructed without any base year and reflects
Financial Action Task Force, are available to cumulative efforts of all stakeholders over the
.u

any resident of that nation. The securities years towards financial inclusion.
w

market regulator of the investors who ● Statement 2 is correct : The index comprises
subscribe should be a member of the three parameters including access, usage and
w

International Organization of Securities quality. It comprises three broad parameters -


Commissions. - access with 35% weightage, usage with 45%
w

weightage and quality with 20% weightage. It

DPP 2023_DAY 188 53


https://upscmaterial.online/
Download From - https://upscmaterial.online/

Contact us :info@onlyias.com

OnlyIAS Nothing Else Visit :dpp.onlyias.in


Contact : +91-7007 931 912

incorporates details of banking, investments, lender and has not used the lender's funds for
insurance, postal as well as the pension sector. the specific objectives for which they were
obtained, instead of diverting the money to
Q.79) Ans: B other uses ; The unit has failed to meet its
Exp: payment/repayment commitments to the
● Statement 1 is not correct : The National lender and has siphoned off the funds, such
Payments Corporation of India (NPCI) serves that the funds have not been used for the
as an umbrella body for the operation of retail precise purpose for which credit was obtained,
payment in India. This organization was nor are the funds available in the form of other
established by the Reserve Bank of India assets with the unit.
along with the Indian Bank’s Association. NPCI ● Laws dealing with Wilful Defaulters : The
was incorporated in December 2008 and was Fugitive Economic Offenders Act 2018: The
centrally promoted by the Reserve Bank of primary goal of this act was to create a
India. The Certificate of Commencement of completely new framework to address such a
Business was issued in April 2009.National situation. The major goal of the Act, according
Payments Corporation of India (NPCI), an to the preamble, is to force 'fugitive economic
umbrella organisation for operating retail offenders to face the law, which is
payments and settlement systems in India, is accomplished by the attachment and
an initiative of Reserve Bank of India (RBI) and subsequent disposition of property.
Indian Banks’ Association (IBA) under the ● Companies Act of 2013: Sections 447 and 448
provisions of the Payment and Settlement of the Companies Act of 2013 would apply to

e
Systems Act, 2007, for creating a robust wilful defaulters. Section 447 states that

n
Payment & Settlement Infrastructure in India. anyone convicted of fraud faces a minimum of

li
● Statement 2 is correct : Considering the utility six months in jail and a maximum of ten years

n
nature of the objects of NPCI, it has been in prison, as well as a fine of three times the
incorporated as a “Not for Profit” Company amount involved in the fraud, or both. False
o
under the provisions of Section 25 of statements are punishable under Section 448.
l.
Companies Act 1956 (now Section 8 of ● Banking Regulation Amendment Act of 2017:
a

Companies Act 2013), to offer improved According to RBI , ‘wilful default' would be
ri

infrastructure for the entire banking industry deemed to have occurred if the unit has
te

to create a robust physical and digital payment defaulted in meeting its payment / repayment
and settlement system. It was created with an obligations to the lender even when it has the
a

intention to provide infrastructure to the capacity to honour the said obligations. The
m

entire Banking system in India for physical as RBI has the authority to provide orders to
well as electronic payment and settlement banks for the resolution of stressed assets
c

systems. under the Banking Regulation Amendment Act


s

of 2017. Furthermore, the RBI has the


p

Q.80) Ans: D authority to appoint authorities or committees


Exp: to assist these banks on how to resolve
.u

● Wilful Defaulter : Any entity is considered a stressed assets. The RBI will approve and/or
w

wilful defaulter when: The unit has failed to appoint the members of these committees.
make its payment/repayment commitments to
w

the lender, despite having the financial means Q.81) Ans: C


to do so ; The unit has failed to meet its Exp:
w

payment/repayment commitments to the

DPP 2023_DAY 188 54


https://upscmaterial.online/
Download From - https://upscmaterial.online/

Contact us :info@onlyias.com

OnlyIAS Nothing Else Visit :dpp.onlyias.in


Contact : +91-7007 931 912

● Option C is correct : Shadow banking is a term and it received presidential approval on 9


used to describe bank-like activities (mainly August 1969.
lending) that take place outside the ● Establishment of National Bank for
traditional banking sector. It is now Agriculture and RuralDevelopment
commonly referred to internationally as non- (NABARD)- 1982 : NABARD came into
bank financial intermediation or market- existence on 12 July 1982 by transferring the
based finance.The shadow banking system is a agricultural credit functions of RBI and
group of financial intermediaries which refinance functions of the then Agricultural
facilitate the creation of credit across the Refinance and Development Corporation
global financial system, but whose members (ARDC). It was dedicated to the service of the
are not subject to regulatory oversight. These nation by the late Prime Minister Smt. Indira
companies are often known as nonbank Gandhi on 05 November 1982.Consequent to
financial companies (NBFCs). The shadow the revision in the composition of share capital
banking system also refers to unregulated between Government of India and RBI,
activities by regulated institutions. Examples of NABARD today is fully owned by the
intermediaries not subject to regulation Government of India.
include hedge funds, unlisted derivatives, and ● Formation of Narasimham Committee II -
other unlisted instruments, while examples of 1998 : The Narasimham Committee II
unregulated activities by regulated institutions (Committee on Banking Sector Reforms) was
include credit default swaps. tasked with conducting a progress review of
● However, the 2008 financial crisis has shown the implementation of banking reforms since

e
that shadow banking can be a source of 1998, with the goal of further strengthening

n
systemic risk to the banking system. The risks India's financial institutions. It concentrated on

li
can be transmitted directly and through the issues such as bank size and capital adequacy

n
interconnectedness of partially-regulated ratios, among other things.
entities with the banking system. After the ● Nationalisation of RBI - 1949 : The Reserve
o
financial crisis, central banks including that of Bank of India was nationalised with effect from
l.
the USA, Britain and the European Union (EU) 1st January, 1949 on the basis of the Reserve
a

have introduced many strong measures to Bank of India (Transfer to Public Ownership)
ri

control shadow banking. In India, the crisis of Act, 1948. All shares in the capital of the Bank
te

the NBFCs that was triggered by the liquidity were deemed transferred to the Central
problems of IL&FS in 2018, has brought back Government on payment of a suitable
a

attention to the shadow banking sector. compensation.


m

Q.82) Ans: A Q.83) Ans: B


c

Exp: Exp:
s

● Nationalisation of the 14 biggest commercial ● Statement 1 is not correct : India Post


p

banks - 1969 : The government through the Payments Bank (IPPB): It was launched on
Banking Companies (Acquisition and Transfer September 1, 2018 under the Department of
.u

of Undertakings) Ordinance, 1969, and Posts, Ministry of Communication . Its 100%


w

nationalized the 14 largest commercial banks equity is owned by the Government of India.
on 19 July 1969. These lenders held over 80 Vision: To build the most accessible, affordable
w

percent of bank deposits in the country. Soon, and trusted bank for the common man in India.
the parliament passed the Banking Companies Fundamental mandate: To remove barriers for
w

(Acquisition and Transfer of Undertaking) Bill, the unbanked and under-banked and reach the

DPP 2023_DAY 188 55


https://upscmaterial.online/
Download From - https://upscmaterial.online/

Contact us :info@onlyias.com

OnlyIAS Nothing Else Visit :dpp.onlyias.in


Contact : +91-7007 931 912

last mile leveraging a network comprising across the country.” Payment enablers,
160,000 post offices (145,000 in rural areas) demand side elements of the payment
and 400,000 postal employees. IPPB’s reach infrastructure, supply side factors of the
and its operating model is built on the key payment infrastructure, payment
pillars of India Stack - enabling Paperless, performance, and customer centricity are the
Cashless and Presence-less banking in a simple five major categories that make up the RBI-DPI.
and secure manner at the customers' ● Option 3 is correct : According to the recent
doorstep, through a CBS-integrated Reserve Bank of India’s (RBI) Report on
smartphone and biometric device. Currency and Finance (RCF), the Indian
● Statement 2 is correct : Fincluvation : It is a economy may take more than a decade to
joint initiative to collaborate with the Fintech overcome the losses caused by the outbreak of
Startup community to co-create and innovate Covid-19 pandemic. The theme of the report is
solutions for financial inclusion. It is the first- "Revive and Reconstruct" in the context of
of-its-kind industry initiative. It would create a nurturing a durable recovery post-Covid-19
powerful platform to mobilize the start-up and rising trend growth in the medium-term
community towards building meaningful
financial products aimed at financial inclusion. Q.85) Ans: C
The country has made rapid strides in the Exp:
FINTECH space in Global Tech world leading ● Statement 1 is correct : According to RBI,
innovations like UPI, Aadhaar. Fincluvation terms loans on which interest or installment
will be a permanent platform of IPPB. of principal remain overdue for a period of

e
more than 90 days from the end of a

n
Q.84) Ans: D particular quarter is called a Non-performing

li
Exp: Asset. NPA refers to a classification for loans or

n
● Option 1 is correct : Financial Stability Report advances that are in default or are in arrears on
(FSR) is a biannual report released by the scheduled payments of principal or interest. In
o
Reserve Bank of India (RBI). The FSR reflects most cases, debt is classified as non-
l.
the collective assessment of the Sub- performing, when the loan payments have not
a

Committee of the Financial Stability and been made for a minimum period of 90 days.
ri

Development Council (FSDC) on risks to ● Statement 2 is not correct : For short duration
te

financial stability, as also the resilience of the crop agriculture loans such as paddy, Jowar,
financial system. As per the report, the Bajra etc. if the loan (installment / interest) is
a

country’s financial system remains stable not paid for 2 crop seasons , it would be
m

despite weakening domestic growth despite termed as a NPA. For Long Duration Crops, the
the risks from global economic uncertainties above would be 1 Crop season from the due
c

and geopolitical developments. date.


s

● Option 2 is correct : Digital Payment Index: To ● Statement 3 is correct : The Securitization and
p

measure the degree of payment digitisation in Reconstruction of Financial Assets and


the nation, the central bank created the Enforcement of Security Interest (SARFAESI)
.u

composite Reserve Bank of India (RBI) – Act has provisions for the banks to take legal
w

Digital Payments Index (RBI-DPI) using March recourse to recover their dues. When a
2018 as the base year. The central bank stated borrower makes any default in repayment and
w

in a press release that “the RBI-DPI Index his account is classified as NPA; the secured
continues to reveal strong growth in creditor has to issue notice to the borrower
w

acceptance and deepening of digital payments giving him 60 days to pay his dues. If the dues

DPP 2023_DAY 188 56


https://upscmaterial.online/
Download From - https://upscmaterial.online/

Contact us :info@onlyias.com

OnlyIAS Nothing Else Visit :dpp.onlyias.in


Contact : +91-7007 931 912

are not paid, the bank can take possession of deposits.The currency deposit ratio (cdr) is
the assets and can also give it on lease or sell the ratio of money held by the public in
it; as per provisions of the SARFAESI Act. currency to that they hold in bank deposits.
cdr = CU/DD. If a person gets Re 1 she will put
Q.86) Ans: B Rs 1/(1 + cdr) in her bank account and keep Rs
Exp: cdr/(1 + cdr) in cash. It reflects people’s
● Option B is correct: Provisioning Coverage preference for liquidity. It is a purely
Ratio (PCR) : A Provisioning Coverage Ratio or behavioural parameter which depends, among
PCR is the percentage of funds that a bank other things, on the seasonal pattern of
sets aside for losses due to bad debts. A high expenditure. For example, cdr increases during
PCR can be beneficial to banks to buffer the festive season as people convert deposits
themselves against losses if the NPAs start to cash balance for meeting extra expenditure
increasing faster. For every loan given out, the during such periods.
banks keep aside some extra funds to cover up ● Statement 2 is not correct : Currency-deposit
losses if something goes wrong with those ratio holds an inverse relationship with the
loans. This is called provisioning.Banks have to money supply. This implies that an increase in
set aside or provide funds to a prescribed currency deposit ratio results in a decrease in
percentage of their bad assets. The the money supply in the economy and vice-
percentage of bad asset that has to be versa. This is because an increase in currency
‘provided for’ is called provisioning coverage deposit ratio implies that people increase
ratio. Provisioning should be made on the basis their cash holdings as compared to the

e
of the classification of assets based on the proportion of deposits held in the bank. This

n
period for which the asset has remained non- implies that banks would have comparatively

li
performing and the availability of security and lesser money to create credit in the economy.

n
the realisable value thereof. For example, if the A reduction in credit creation will lead to a fall
provisioning coverage ratio is 70% for a in money supply prevailing in the economy.
o
particular category of bad loans, banks have to The vice-versa of this process will be applicable
l.
set aside funds equivalent to 70% those bad in case of a fall in currency deposit ratio.
a

assets out of their profits. Assets of a bank


ri

means loans they have given and investment Q.88) Ans: A


te

they have made. If the loans are not coming Exp:


back, there should be provisioning for such bad ● Statement 1 is not correct: Fiat money is
a

debts. The assets are classified by the RBI in government-issued currency that is not backed
m

terms of their duration of non-repayment by a physical commodity, such as gold or silver,


(NPA, doubtful asset etc.). The primary but rather by the government that issued it.
c

responsibility for making adequate provisions The value of fiat money is derived from the
s

for any diminution in the value of loan assets, relationship between supply and demand and
p

investment or other assets is that of the bank the stability of the issuing government, rather
managements and the statutory auditors. than the worth of a commodity backing it as is
.u

the case for commodity money. Most modern


w

Q.87) Ans: A paper currencies are fiat currencies, including


Exp: the U.S. dollar, the euro, and other major
w

● Statement 1 is correct: The currency deposit global currencies. Fiat money has no intrinsic
ratio shows the amount of currency that value, while legal tender is any currency
w

people hold as a proportion of aggregate declared legal by a government. Governments

DPP 2023_DAY 188 57


https://upscmaterial.online/
Download From - https://upscmaterial.online/

Contact us :info@onlyias.com

OnlyIAS Nothing Else Visit :dpp.onlyias.in


Contact : +91-7007 931 912

can issue fiat currency and make it legal tender bank and that a review of the framework
by setting it as the standard for debt would result in excess capital being freed,
repayment.Governments can issue fiat which the RBI can then share with the
currency and make it legal tender by setting it government. The government believes that RBI
as the standard for debt repayment. The is sitting on much higher reserves than it
benefit of fiat money is that it gives central actually needs to tide over financial
banks greater control over the economy, but emergencies that India may face. Some central
governments can print too much money and banks around the world (like US and UK) keep
create hyperinflation. The U.S. dollar , Indian 13% to 14% of their assets as a reserve
Rupee etc are both fiat money and legal compared to RBI’s 27% and some (like Russia)
tender. more than that. Economists in the past have
● Statement 2 is correct : Legal tender is argued for RBI releasing ‘extra’ capital that can
anything recognized by law as a means to be put to productive use by the government.
settle a public or private debt or meet a The Malegam Committee estimated the excess
financial obligation, including tax payments, (in 2013) at Rs 1.49 lakh crore.
contracts, and legal fines or damages. Legal ● In the past, the issue of the ideal size of RBI’s
tender is something which is acknowledged by reserves was examined by three committees
the laws as a mechanism to settle a private or – V Subrahmanyam (1997), Usha Thorat
public debt or in order to meet a fiscal (2004) and Y H Malegam (2013). The current
responsibility which includes paying taxes, committee headed by Bimal Jalan is also
abiding by contracts, and finally damages or charged with similar terms of reference. The

e
fines. Almost every country uses its national Subrahmanyam committee recommended

n
currency as legal tender. Creditors are lawfully that the contingency reserve should be built up

li
responsible for accepting legal tender for the to 12 per cent, the Thorat committee said the

n
repayment of debt that they have availed. reserve adequacy should be maintained at 18
Legal tender is constitutional by a law that per cent of the total assets. The YH Malegam
o
specifies the object to be utilised as legal committee recommended that till such time
l.
tender and the organisation that is the reserves were considered “excess” the
a

commissioned to create and issue the same to complete surpluses should be transferred to
ri

the public such as the Reserve Bank of India. the government as dividends. Following the
te

recommendations of the Malegam committee


Q.89) Ans: C in the year 2013-14 RBI transferred its entire
a

Exp: surplus to the Government of India. This was a


m

● Option C is correct : Economic capital record amount of Rs.526.79 billion. In the four
framework refers to the risk capital required years preceding RBI had transferred around
c

by the central bank while taking into account 52% of its total surplus to the government.
s

different risks. The economic capital


p

framework reflects the capital that an Q.90) Ans: A


institution requires or needs to hold as a Exp:
.u

counter against unforeseen risks or events or ● A central bank balance sheet is a reflection of
w

losses in the future. Existing economic capital its various functions, particularly its role as a
framework which governs the RBI’s capital monetary authority and as banker to the
w

requirements and terms for the transfer of its Government and banks. The structure of assets
surplus to the government is based on a and liabilities of the Reserve Bank are, more or
w

conservative assessment of risk by the central less, in line with the balance sheet followed by

DPP 2023_DAY 188 58


https://upscmaterial.online/
Download From - https://upscmaterial.online/

Contact us :info@onlyias.com

OnlyIAS Nothing Else Visit :dpp.onlyias.in


Contact : +91-7007 931 912

most central banks. The accounts of the setting up of the Industrial Finance
Reserve Bank are, however, bifurcated into the Corporation (IFCI). Subsequently, India’s
Issue Department, reflecting the currency Industrial Credit and Investment Corporation
issue function and the Banking Department, (ICICI) was set up with the World Bank’s
accounting for all other central banking backing in 1955. The Industrial Development
functions (such as banker to the Government Bank of India (IDBI) came into existence in 1964
and banks) in terms of Section 23(1) of the to promote long-term financing for
Reserve Bank of India Act, 1934, following the infrastructure projects and industry.
recommendations of the Hilton Young ● Statement 2 is correct : The prime objective of
Commission (1926). However, the table DFI is the economic development of the
provided in WSS is an addition of assets and country. These banks provide financial as well
liabilities of both departments in such a way as the technical support to various sectors DFIs
that it gives the clear picture to the market do not accept deposits from people . They
participants and other stakeholders. It is one of raise funds by borrowing funds from
the most important tables by which we can governments and by selling their bonds to the
understand RBI’s interactions with the general public. It also provides a guarantee to
financial system on a weekly basis banks on behalf of companies and
● Liabilities of Reserve Bank : Notes Issued, subscriptions to shares, debentures, etc.
Notes in Circulation, Deposits, Deposits of the
Central Government, Deposits of the State Q.92) Ans: C
Government, Market Stabilisation Scheme , Exp:

e
Deposits of Scheduled Commercial Banks, ● Statement 1 is correct : National Asset

n
Deposit of Scheduled State Co-operative Reconstruction Company Ltd.(NARCL), India’s

li
Banks, Deposits of Other Banks, etc first-ever Bad Bank, was set up in 2021, and RBI

n
● Assets of Reserve Bank : Foreign Currency has recently granted the same under the
Assets, Gold Coin Bullion, Rupee Securities, SARFAESI Act 2002. If the bad bank is unable to
o
Loans and Advances to central and state sell the bad loan or has to sell it at a loss, then
l.
governments, Loans and Advances to SCBs, the government guarantee will be invoked.
a

State Co-operative Banks etc NARCL has been incorporated under the
ri

Companies Act and has applied to Reserve


te

Q.91) Ans: A Bank of India for license as an Asset


Exp: Reconstruction Company (ARC). NARCL has
a

● Development financial institutions provide been set up by banks to aggregate and


m

long-term credit for capital-intensive consolidate stressed assets for their


investments spread over a long period and low subsequent resolution. PSBs will maintain51%
c

yielding rates of return, such as urban ownership in NARCL.


s

infrastructure, mining and heavy industry, and ● Statement 2 is correct : Setting up of NARCL,
p

irrigation systems. Development banks are the proposed bad bank for taking over stressed
different from commercial banks, which assets of lenders, was announced in the
.u

mobilize short- to medium-term deposits and Budget for 2021-22. The plan is to create a bad
w

lend for similar maturities to avoid a maturity bank to house bad loans of ₹500 crore and
mismatch (a potential cause for a bank’s above, in a structure that will contain an asset
w

liquidity and solvency). reconstruction company (ARC) and an asset


● Statement 1 is not correct : In India, the first management company (AMC) to manage and
w

DFI was operationalized in 1948 with the recover dud assets. This new entity is being

DPP 2023_DAY 188 59


https://upscmaterial.online/
Download From - https://upscmaterial.online/

Contact us :info@onlyias.com

OnlyIAS Nothing Else Visit :dpp.onlyias.in


Contact : +91-7007 931 912

created in collaboration with both public and Wealth Holders: They stand to lose due to
private sector banks. inflation, as their real returns fall due to rise in
prices.
Q.93) Ans: B
Exp: Q.94) Ans: B
● In economics, inflation (or less frequently, Exp:
price inflation) is a general rise in the price ● Statement 1 is correct : A type of debt
level of an economy over a period of time. instrument that is not secured by physical
When the general price level rises, each unit assets or collateral. Debentures are backed
of currency buys fewer goods and services; only by the general creditworthiness and
consequently, inflation reflects a reduction in reputation of the issuer. Both corporations
the purchasing power per unit of money – a and governments frequently issue this type of
loss of real value in the medium of exchange bond in order to secure capital. Like other
and unit of account within the economy.The types of bonds, debentures are documented in
effect of inflation is not distributed evenly in an indenture. In Other Words , A debenture is
the economy. There are chances of hidden a document that either creates a debt or
costs for different goods and services in the acknowledges it, and it is a debt without
economy. Sudden or unpredictable inflation collateral. In corporate finance, the term is
rates are harmful to an overall economy. They used for a medium- to long-term debt
lead to market instability and thereby make it instrument used by large companies to borrow
difficult for companies to plan a budget for the money. In some countries the term is used

e
long-term. interchangeably with bond, loan stock or note.

n
● Effects of Inflation on Different Sections : ● Statement 2 is not correct : Debenture holders

li
Creditor/lender: Inflation harms creditors, as (investors) do not have any rights to vote in

n
they lose in real terms. A 1000 RS lent @ 5%, the company’s general meetings of
will pay an interest rate of 50. If inflation rises shareholders, but they may have separate
o
to 10%, the price of goods will be 1100, but meetings or votes e.g. on changes to the rights
l.
after interest, the return will only be 1050. attached to the debentures. The interest paid
a

Debtor/Borrower : Inflation benefits the to debenture holders is calculated as a charge


ri

Debtor as they gain in real terms. A basic rule against profit in the company’s financial
te

of inflation is that it causes the value of a statements.


currency to decline over time. In other words, ● Statement 3 is correct : The main advantage of
a

cash now is worth more than cash in the debentures to companies is the fact that they
m

future. Thus, inflation lets debtors pay lenders have a lower interest rate than e.g. overdrafts.
back with money worth less than it was when Also, they are usually repayable at a date far
c

they originally borrowed it.Pensioners : off in the future. For an investor, their main
s

Inflation harms the pensioners, if their advantages are that they are often easy to sell
p

pensions are not indexed to inflation, and in stock exchanges and they contain less risk
loses money. Inflation harms pensioners as than e.g. equities.
.u

they generally live on a fixed income consisting


w

of Social Security, pensions and withdrawals Q.95) Ans: C


from their retirement portfolios. Producers : Exp:
w

They stand to gain by inflation since the price ● Statement 1 is not correct : Bombay Stock
of goods and services rise faster than the cost Exchange (BSE), which was established in
w

of production as wages take time to react. 1875, is Asia’s oldest stock exchange and it

DPP 2023_DAY 188 60


https://upscmaterial.online/
Download From - https://upscmaterial.online/

Contact us :info@onlyias.com

OnlyIAS Nothing Else Visit :dpp.onlyias.in


Contact : +91-7007 931 912

was the first stock exchange in India. Surely discounting, trading and settling invoices / bills
enough, the trading was extensive even of MSMEs and facilitating both receivables as
then.BSE did not have on-screen trading. The well as payables factoring (reverse factoring).
government founded National Stock Exchange MSME sellers, corporate and other buyers,
(NSE) in 1992 and the trading on the exchange including Government Departments and PSUs,
began in 1994. The National Stock Exchange and financiers (banks, NBFC-Factors and other
(NSE), with its own benchmark index named financial institutions, as permitted) are direct
Nifty, introduced on-screen trading. This participants in the TReDS and all transactions
grabbed traders’ attention as it helped them in processed under this system are ‘”without
saving the brokers’ costs. Later, BSE too started recourse” to MSMEs.
on-screen trading. Though most of the ● Statement 2 is not correct : Only MSMEs can
companies are listed on both the stock participate as sellers in TReDS. Corporates,
exchanges, the stock prices of the companies Government Departments, PSUs and any
could be different. The number of companies other entity can participate as buyers in
listed on BSE is more than the ones listed on TReDS. Banks, NBFC – Factors and other
National Stock Exchange (NSE). financial institutions as permitted by the
● Statement 2 is not correct : The SEBI is the Reserve Bank of India (RBI), can participate as
regulatory authority formed under Section 3 financiers in TReDS.
of the SEBI Act 1992 to safeguard the interests
of investors in securities, promote the Q.97) Ans: D
development of and regulate the securities Exp:

e
market, and for matters associated with and ● Statement 1 is not correct : International

n
incidental to the securities market. The Financial Services Centres Authority (IFSCA) :

li
Securities and Exchange Board of India (SEBI) is The IFSCA was established in April 2020 under

n
the regulatory body for securities and the International Financial Services Centres
commodity markets in India under the Authority Act, 2019. It is a statutory authority
o
ownership of the Ministry of Finance within established by the Indian Government. It is an
l.
the Government of India. It was established on authority to develop and regulate financial
a

12 April 1988 as an executive body and was services, financial products and financial
ri

given statutory powers on 30 January 1992 institutions in the International Financial


te

through the SEBI Act, 1992. Services Centre (IFSC) in India. Currently,
there is only one IFSC being developed in India,
a

Q.96) Ans: A Gift City, in Gandhinagar, Gujarat. Before the


m

Exp: IFSCA was established, the financial services


● Statement 1 is correct : TReDS is an electronic and institutions were regulated by the
c

platform for facilitating the financing / domestic financial regulators such as SEBI, RBI,
s

discounting of trade receivables of Micro, IRDAI, PFRDA, etc. The main goal of the IFSCA
p

Small and Medium Enterprises (MSMEs) is to promote ease of doing business in IFSC
through multiple financiers. These receivables and provide a world class regulatory
.u

can be due from corporates and other buyers, environment.


w

including Government Departments and Public ● Statement 2 is not correct : An International


Sector Undertakings (PSUs). TReDS is a Financial Services Centre (IFSC) is a financial
w

payment system authorised under the centre that caters to customers outside the
Payment and Settlement Systems (PSS) Act, jurisdiction of the domestic economy. It is also
w

2007. It is a platform for uploading, accepting, known as an offshore financial centre since it

DPP 2023_DAY 188 61


https://upscmaterial.online/
Download From - https://upscmaterial.online/

Contact us :info@onlyias.com

OnlyIAS Nothing Else Visit :dpp.onlyias.in


Contact : +91-7007 931 912

deals with flow of finance, financial products Q.99) Ans: A


and services across borders. An IFSC is, thus, a Exp:
jurisdiction that provides world class financial ● Option A is correct : Blue chip stocks are
services to non-residents and residents, to the shares of very large and well-recognised
extent permissible under the current companies with a long history of sound
regulations, in a currency other than the financial performance. These stocks are
domestic currency of the location where the known to have capabilities to endure tough
IFSC is located. market conditions and give high returns in
good market conditions. Blue chip stocks
Q.98) Ans: B generally cost high, as they have good
Exp: reputation and are often market leaders in
● Statement 1 is correct : Insurance is a legal their respective industries. A blue-chip
agreement or contract between an insured company is one that has a national reputation
and insurer that offers protection against any for quality, reliability, and profitability in both
kind of loss with the latter compensating for good and bad market conditions. These
the loss the former has incurred. Both the Life businesses have a good financial history and a
Insurance and the Non-life Insurance is proven track record of creating profits with
regulated by the IRDAI (Insurance Regulatory only a small amount of debt. These businesses
and Development Authority of India). It is an are considered stable since they have been in
Autonomous body, setup under the IRDA Act, operation for a long time. Examples of Blue
1999, on the recommendations of R N Chip: Blue-chip companies include Bharti Airtel

e
Malhotra committee. It is tasked with the Ltd., BPCL, Dr Reddy, HDFC, Reliance Industries

n
regulation of the insurance sector in India. Ltd., Infosys Technologies Ltd., and ICICI Bank

li
● Statement 2 is not correct : Insurance Ltd.

n
penetration is measured as the percentage of ● The following are the features of these
insurance premium to GDP. 2% is India’s companies : Assured Returns: Blue-chip stocks
o
insurance penetration compared to the global provide guaranteed profits in the form of
l.
average of 7.4%. Insurance density is dividends, which are paid out quarterly. The
a

calculated as the ratio of premium to fact that well-established enterprises serve as


ri

population (measured in US$ for convenience a safe investment option for most investors.
te

of international comparison). The insurance Creditworthiness: Blue-chip corporations have


density in India increased from $11.5 in 2001 sufficient money to easily meet their financial
a

to $78 in 2020. As of 2020, the penetration for obligations. As a result, the creditworthiness of
m

life insurance in India is 3.2 percent and non- the shares issued by such enterprises is high.
life insurance penetration is 1 percent. While Risk factor: Blue-chip stocks have a lower risk
c

India is at par with international average in factor than other types of equities. Investing
s

terms of insurance penetration for life horizon: Most investments are made over a
p

insurance, we lag behind in terms of non-life period of seven years or more. Because of their
insurance. Globally, insurance penetration was long investment horizon, blue-chip stocks are
.u

3.3 per cent for the life segment and 4.1 per ideal for accomplishing long-term financial
w

cent for the non-life segment in 2020. Average goals. Growth Prospect: Blue-chip firms are
growth rate of the insurance sector in India is major corporations that have reached the
w

around 12%. Mortality protection gap in the pinnacle of their growth potential. This has an
insurance sector in India is 83%, indicating the impact on India's blue-chip stocks, which have
w

huge potential of this sector. been steadily increasing over time. Taxation:

DPP 2023_DAY 188 62


https://upscmaterial.online/
Download From - https://upscmaterial.online/

Contact us :info@onlyias.com

OnlyIAS Nothing Else Visit :dpp.onlyias.in


Contact : +91-7007 931 912

In India, profits made from blue-chip stocks are Market is one where the securities are traded
taxed as income under Section 80 C of the for immediate delivery and payment, whereas
Income Tax Act. a Future Market is one where the securities are
traded for future delivery and payment. There
Q.100) Ans: C is another variant of the secondary market
Exp: which is known as ‘Options Market’. In this
● Statement 1 is correct : Primary Market is that type of secondary market, the trading of the
segment of the capital market which deals securities is for conditional future delivery.
with new securities. In the primary market, They are also of two types: Option and Put
the securities are created. This means that the Option.
new securities are offered for sale for the first ● Statement 3 is not correct : Primary Market is
time in this type of market. Hence it is also that form of capital market where the
known as ‘new issues market’. The main securities are issued for the first time to the
function of the primary market is to facilitate investors by the company. On the other hand,
companies in order to raise long term funds by the secondary market is the market where
making new issues of the shares of ventures. already-issued securities are traded between
So in this market, the company sells directly to buyers and sellers through a stock exchange. In
the investors. It also includes further capital a primary market, the company is directly
issues by the companies, whose shares are involved in the transaction whereas, in the
previously quoted on the stock exchange. secondary market, there is no involvement of
There can be an alternate definition of the the company, since the transaction occurs

e
primary market as well. This says that a between the investors. In the case of a primary

n
primary market is a place: where an initial market, a the company is directly involved in

li
public offering takes place ; Government or the transaction, the gain/loss belongs to the

n
public sector institutions raise money through company itself. But in case of a secondary
bond offerings. market, the gain/loss belongs to the investors.
o
● Statement 2 is correct : The secondary
l.
market, or otherwise called the stock market,
a

is a place where the securities are traded


ri

between investors, after their issue to the


te

public in the primary market. In other words,


it is a market wherein the investor purchases
a

securities from another investor. Hence the


m

loss or the gain belongs to the investor and not


to the company. It includes both the equity
c

market and the debt market. Further, the


s

secondary market can be in the form of a stock


p

exchange or over the counter market.Any


transaction happening after the IPO falls under
.u

the secondary market. The major exchanges of


w

the market which deal with the secondary


exchange are NYSE, NASDAQ, FTSE, HANG
w

SENG, BSE and NSE etc. There are two


components of the secondary market. They
w

are the Spot Market and Future Market. A Spot

DPP 2023_DAY 188 63


https://upscmaterial.online/

You might also like